You are on page 1of 143

Exam 1

(1) A nurse is caring for a client admitted 2days ago with myocardial infection .upon
assessment the nurse notes a new systolic murmur at the cardiac apex. The nurse
should assess the client for which of the following conditions?
(A) A. Ventricular aneurysm
(B) Acute pericarditis
(C) Papillary muscle dysfunction
(D) Pulmonary embolism

2. Which one of the following is an u


​ ncontrollable​ risk factor that has been linked to
development of coronary artery disease(CAD)
(A) Exercise
(B) obesity
(C) stress
(D) heredity
Rationale:
Uncontrollable risks are factors that can not be modified like genetics, heredity, age
Controllable factors are those that are modifiable like - lifestyle(exercise, nutrition),
obesity and stress.
3. A 21 year old male client is transported by ambulance to the emergency unit. He complains of
severe pain in his right chest where he struck the steering wheel. Which is the primary client
goal at this time?
A. Reduce client’s anxiety
​B. Maintain adequate oxygenation
C. Decrease chest pain
D. Maintain adequate circulating volume
4. The following finding is suggestive of a pneumothorax in a patient with chest trauma;
(A) absent breath sounds
(B) inspiratory wheezing
(C) dullness on chest percussion
(D) pronounced crackles

Rationale:
Pneumothorax - collapsed lung due to changes in pressure within pleural space and
chest wall, causing air to leak out. Leading to a collapsed lung, no ventilation hence
absent breath sounds.

(5) The primary reason for administering digoxin (Lanoxin) to a patient is to,
(A) Relax the walls of the heart’s arteries.
(B) Improve the strength of the heartbeat.
(C) Prevent irregularities in ventricular contractions .
(D) Decrease inflammation of the heart wall.

Rationale :

6) The priority nursing diagnosis for a patient with heart failure and pulmonary oedema is,
(A) Risk for infection related to stasis of alveolar secretions
(B) Impaired skin integrity related to pressure
(​C) Activity intolerance related to pump failure
D)Constipation related to immobility

(7) When preparing a patient for colonoscopy procedure the nurse should,
(A) Perform a high colonic washout procedure the night before the procedure
(B) Ensure the patient takes a high roughage diet with a laxative the night before
procedure
(C) Educate the patient to report any “needle prick” sensations within 12 hrs after the
procedure
(D) Advise the patient to take low fiber diet for 1 week before the procedure

8) The characteristics of a “Cushingoid” appearance in Cushing’s disease include,


(A) Hypotension, Kussmauls’ breathing, hunchback & diaphoresis
(B) Moon face, buffalo hump, central obesity & thin musculature
(C) Mood swings, slow wound healing, hypertension & tunnel vision
(D) Flushing, brittle skin, abdominal pain and encephalopathy

Rationale:
In women, it can cause menstrual irregularities(amenorrhea, no periods or
menorrhagia-heavy period or irregular periods. Also, infertility, hypertrophy of the
clitoris, atrophy of vaginal muscles, deepening of voice and decreased libido may
occur in women related to inhibition of pulsatile secretion of luteinizing hormone (LH)
and follicle-stimulating hormone (FSH), which likely is due to interruption of
luteinizing hormone-releasing hormone (LHRH) pulse generation. In men, inhibition of
LHRH and FSH/LH function may lead to decreased libido and impotence. Can cause
baldness in med

(10) The sensitivity part of a culture & sensitivity test is for the purpose of,
(A) Identifying the causative organism for the infection diagnosed
(B) Identifying the most effective antibiotics against the causative organism
(C) Identifying the antibiotics that the patient is resistant to
(D) Testing the concentration of antibiotic required to cure an infection 1

(11) A client with a cataract would most likely complain of,


(A) Halos and rainbows around lights
(B) Eye pain and irritation that worsens at night
(C) Blurred and hazy vision
(D) Eye strain and headache when doing close work

12) The primary rationales for tube feeding for a client post total laryngectomy due to
cancer of larynx is to,
(A) Meet dietary needs and prevent fistula formation
(B) Maintain an open airway and prevent aspiration
(C) Meet fluid & dietary needs and prevent aspiration
(D) Prevent fistula formation and reduce aspiration

13) When a burn occurs in an adult,


(A) Using rule of nine, right arm is calculated as being 18% & right leg 9%.
(B) Capillaries & small vessels dilate, renal blood flow reduces
(C) Fluid shifts from ICF to ECF space/compartment along with potassium.
(​D) Permeability of the kidney tubules increases with oliguria

(14) The following technique should be avoided when changing a hemiparalysis clients’
position in bed;
(A) Sliding the client to move her up in bed
(B) Rolling the client onto her side
(C) Lifting the client when moving her up in bed
(D) Having the client lift herself off the bed using a trapeze

15) When a blood pressure (BP) reading is abnormal upon initial assessment, it is best to
check the BP,
(A) a) On the other arm
(B) b) On both arms sitting and standing
(C) c) With the patient standing
(D) d) After a 5 min wait

(16) Swabs, instruments and sharps count during surgery is carried out by,
(A) Circulating and anaesthetic nurses
(B) Anaesthetic and scrub nurses
(C) Circulating and Scrub nurse​s
(D) Recovery room and scrub nurses

17) When nursing a patient with chronic obstructive pulmonary disease, the nurse should,
(A) Ensure the patient gets high flow rate oxygen at 100%
(B) Advise the patient to get influenza and pneumococcal vaccines
(C) Avoid administration of prophylactic bronchodilators
(D) Position the patient in dorsal recumbent position to enhance ventilation

(18) After a clients’ nasogastric tube has been removed the nurse should,
(A) Offer the client liquids to drink
(B) Provide the client with oral hygiene
(C) Encourage client to cough and deep breath
(D) Auscultate the client’s bowel sounds

(19) Neurovascular assessment of the lower limb post-cast application involves


observation of,
(A) Color, sensation and pulses
(B) Level of consciousness, limb symmetry and muscular strength
(C) Capillary refill, sensation and muscle power
(D) Color, warmth and muscle power

20) The risk for macrovascular complications in diabetes mellitus may be reduced by,
(A) Lowering BP, avoiding nephrotoxic substances & reducing strenuous exercises.
(B) Weight reduction, reducing fat/triglyceride levels, quitting smoking.
(C) Foot care, early diagnosis of retinal detachment & daily U/Es checkups.
(​D) High fibre diet, low fat diet & prompt treatment of systemic infections

(21) The priority indicator of increasing intracranial pressure is,


(A) Unequal pupil size
(B) decreasing systolic pressure
(C) tachycardia
(D) decreasing body temperature

(22) Foods that may predispose a client to megaloblastic anemia if taken exclusively
include,
(A) Meat, eggs and beans
(B) Beans, Peas and rice
(C) Capsicum, Sorghum and Millet
(D) Meat, Maize, Sorghum

23) In pyloric stenosis there is,


(A) a) Projectile vomiting & olive-sized mass in abdomen’s upper right quadrant
(B) b) Blood tinged vomitus and significantly reduced peristaltic waves
(C) c) Severe abdominal pain and irregular opening of the bowels
(D) d) Normal feeding appetite and passing of loose bowel motion

24) The following are risk factors for impaired gas exchange and pulmonary infection :
(A) Immobility, constipation and incontinence
(B) Fluid restriction, immunosuppression and decreased level of consciousness.
(C) Fluid restriction, immobility and ingestion of seasoned foods
(D) Incontinence, chemotherapy and anemia

(25) Buck’s traction is,


(A) a) A form of skin traction and is often used to relieve muscle spasms.
(B) b) A form of skeletal traction for compound fractures.
(C) c) Only used in correction of hip and femur fractures.
(D) d) Always contra-indicated in obese ju patients.

(26) Open angle glaucoma is mainly treated using,


(A) a) Proton pump inhibitors, quinolones and glycosides
(B) b) Antibiotics, osmotic diuretics and haematinics
(C) c) Miotics, Carbonic anhydrase inhibitors and epinephrine
(D) d) Stool softeners, anti-hypertensives and Miotics

27) When nursing a patient on steroids, the nurse should,


(A) a) Observe for oedema, weigh daily, monitor blood counts
(B) b) Maintain infection control, monitor blood sugar, maintain hydration
(C) c) Observe for bone marrow suppression, minimise exposure to heat,encourage
bed rest
(D) d) Encourage increased fluid intake, administer anti-emetics, monitor RBC count

(28) In acute heart failure with pulmonary edema, morphine is administered in small doses
to,
(A) a) Promote excretion of sodium and water by kidneys
(B) b) Relieve hypoxemia and dyspnea
(C) c) Delay release of calcium from intracellular reservoirs
(D) d) Reduce peripheral resistance & venous return3

(29) The common causative organism for pharyngitis is,


(A) a) Group B, beta hemolytic streptococcus
(B) b) Group A, beta hemolytic streptococcus
(C) c) Hemophilus influenza
(D) d) Staphylococcus aureus

30) In pulmonary embolism (PE),


(A) a) There is a clot formation in the pulmonary vein and or its branches
(B) b) There is obstruction of the Pulmonary artery or one of its branches
(C) c) Trauma, pregnancy and Valsava maneuvers are common predisposing factors
(D) d) Old age, heart failure and intestinal obstruction clients are more prone

31) The purpose of monitoring central venous (CVP) for the critically ill patients is to,
(A) a) Help assess the functioning of left side of the heart
(B) b) Monitor fluid replacement therapy
(C) c) Estimate the levels of pulmonary pressures in the critically ill
(D) d) Help assess functioning of the mitral and tricuspid valves

32) The antidote for heparin is,


(A) a) Vitamin K
(B) b) Warfarin (Coumadin)
(C) c) Thrombin
(D) d) Protamine sulfate

33) The frontal lobe of the brain is,


(A) a) The primary receptive area for vision
(B) b) Mainly responsible for auditory sensation and coordination
(C) c) Centre for respiratory and vasomotor activity
(D) d) Responsible for abstract thinking, judgment and emotions

34) Dumping syndrome is as a result of,


(A) a) Rapid emptying of hypertonic food from the stomach into the jejunum
(B) b) Failure of a patient with a peptic ulcer to eat regularly
(C) c) Chronic anemia among the elderly patients
(D) d) Loss of the large intestine functionality following hemicolectomy

35) After cataract surgery, if a patient complains of nausea, the nurse should,
(A) a) Instruct patient to perform deep breaths until the nausea reduces
(​B) b) Administer antiemetic to prevent intraocular pressure elevation
(C) c) Ensure a vomiting bowl is placed close to the patient as possible
(D) d) Reassure the patient that it is a common feeling post general Anaesthesia.

(36) Immediately postoperatively a nurse should,


(A) a) Position a patient in recovery position, measure urine output and assess
respirations every 15 minutes
(​B) b) Maintain patent airway, measure central venous pressure (CVP) and check
oxygen saturation levels
(C) c) Position patient in recovery position, observe adverse effects of anesthesia and
assess reflexes
(D) d) Check IV site for infiltration, administer clear oral fluids and monitor vital signs
every 15 minutes
(39) The causes of pre-renal failure include,
(A) a) Renal caliculi, benign prostatic hypertrophy and blood clots
(B) b) Tubular necrosis, septic shock and hemolytic blood reaction transfusion
(C) c) Hemorrhage, burns and dehydration
(D) d) Tumours, Trauma and chemical toxicity

(40) For a patient to undergoing a kidney, ureter and bladder X-ray is important for the
nurse to know that,
(A) a) There is no special preparation required.
(B) b) Oral fluids must be withheld the morning of the examination.
(C) c) The patient should have a full bladder before the procedure.
(D) d) An enema should be administered before the procedure

(41) Treatment of megaloblastic anemia includes;


(A) a) Chemotherapy, radiotherapy, hydration
(B) b) Oxygen therapy, hydration and iron supplements
(C) c) Blood transfusion, iron supplements, splenectomy
(​D) d) Oxygen therapy, folate supplements, Alcohol restriction

(42) For a patient with cellulitis the nurse should,


(A) A. Apply warm compresses to the site and elevate affected extremity
(B) B. Apply cold compresses to the site and elevate affected extremity
(C) C. Administer topical antibiotics and oral analgesics as prescribed
(D) D. Administer plasma expanders and elevate the affected limb to 90o

43) The vitamin required for normal production of RBCs is;


(A) calcium
(B) vitamin A
(C) folic
(D) vitamin K

(44) Plain insulin has peak action of;


(A) 8- 12hours
(B) 2-3 hours
(C) 12-16hours
(D) 16-30hours
45) Valsava maneuver can result to bradycardia. Which of the following activities will not
stimulate valsalva’s maneuver?
(A) a) Use of stool softeners
(B) b) Enema administration
(C) c) Gaggling while tooth brushing
(D) d) Lifting heavy objects

46) Which of the most relevant knowledge about oxygen administration to a client with
COPD
(​A) a) Oxygen at 1-2L/min is given to maintain the hypoxic stimulus for breathing.
(B) b) Hypoxia stimulates the central chemoreceptors in the medulla that makes the
client breath.
(C) c) Oxygen is administered best using non-rebreathing mask.
(D) d) Blood gases are monitored using a pulse oximeter.

47) What is the reason for the nurse in instructing the client to rotate the injection sites for
insulin?
(A) a) lipodystrophy can result and extremity painful
(B) b) Poor rotation technique can cause superficial hemorrhaging
(C) c) lipodystrophic areas can result, causing erratic insulin absorption rates from
these
(D) d) Injection sites can never be reused

48) Which of the following is included in Orem’s theory


(A) a) Maintenance of a sufficient intake of air
(B) b) Self-perception
(C) c) Love and belonging
(D) d) Physiologic needs

(49) The patient with Angina pectoris is being discharged home with nitroglycerine tablets
which of the following instructions does the nurse include in the teaching?
(A) a) ”when the chest begins,place the tablet under the tongue .if it continues take
another tablet in 5 minutes.”
(B) b)”place the tablet under the tongue,if the pain is not relieved in 15 minutes go to
hospital”
(C) c) ccontinue your activity and if the pain does not go away in10 minutes,begin
taking the nitroglycerine tabs for 15 minutes then go and lie”
(​D) d) Place the nitroglycerine tab under the tongue every 5 minutes for 3 doses.Go to
hospital if the pain is unrelieved

(50) It is best to describe as a systematic, rational method of planning& providing nursing


care for individuals, families, group and community.
(A) assessment
(B) nursing process
(C) diagnosis
(D) implementation

(51) A client has just undergone spinal fusion after experiencing herniation of a lumbar
disk. The nurse would avoid which intervention to maintain client safety after this
procedure?
(A) Use the overhead trapeze.
(B) Keep the head of the bed flat.
(C) Place pillows under the length of the legs.
(D) Use logrolling technique for repositioning.

(52) Nurse Nyaruai is giving a health education talk to Mothers attending the MCH/FP
clinic at Kapkoros health center. As a knowledgeable nurse, she must know that a mother
who breastfeeds her child passes on which antibody through breast milk?
(A) IgA
(B) IgG
(C) IgM
(D) IgE

(53) Which of the following interventions would be helpful to a breastfeeding mother who
is experiencing engorged breasts?
(A) A. Applying ice
(B) B. Administering bromocriptine
(C) C. Applying a breast binder
(D) D. Teaching how to express her breasts in a warm shower
(54) You are preparing to perform a fundal assessment on a postpartum client. The initial
nursing action in performing this assessment is which of the following?
(A) A. Ask the client to turn on her side
(B) B. Ask the mother to urinate and empty her bladder
(C) C. Massage the fundus gently before determining the level of the fundus.
(D) D. Ask the client to lie flat on her back with the knees and legs flat and straight

55) Which of the following complications is Most likely responsible for a delayed
postpartum hemorrhage?
(A) A. Cervical laceration.
(B) B. Uterine sub involution
(C) C. Perineal laceration
(D) D. Clotting deficiency

56) Following a difficult labor and birth, a primiparous woman refuses to feed her baby
four hours after delivery, she states that she is too tired and just wants to sleep. The nurse
should:
(A) A. Tell the woman she can rest after she feeds her baby
(B) B. Take the baby back to the nursery, reassuring the woman that her rest is a
priority at this time
(C) C. Recognize this as a behavior of the taking-hold stage
(D) D. Record the behavior as ineffective maternal-newborn attachment

57) MrsWangombe is under your care and expresses concern about the weight she gained
during pregnancy and how quickly she can lose it now that the baby is born. In describing
the expected pattern of weight loss, you should begin by telling her that:-
(A) A. Lactation will inhibit weight loss since caloric intake must increase to support
milk production
(B) B. Return to pre pregnant weight is usually achieved by the end of the postpartum
period
(C) Fluid loss from diuresis, diaphoresis, and bleeding accounts for about a 3 pound
weight loss
(D) D. The expected weight loss immediately after birth averages about 11 to 13
pounds

(58) Signs of puerperal sepsis include


(A) A. Pyrexia, tachycardia, whitish lochia
(B) A. Pyrexia, tachycardia, red lochia
(C) A. Pyrexia, tachycardia, pink lochia
(D) A. Pyrexia, tachycardia, offensive lochia

59) Systemic changes during normal puerperium include


(A) A. Involution of the uterus, lochia loss, blood volume returns to normal
(B) B. Rising of the fundus, lochia loss, blood volume decreases
(C) C. Involution of the uterus, lochia loss, blood volume increases
(D) D. Maternal weight gain

(60) About puerperium, the following statement is true


(A) A. The puerperium is a period of 60 days following delivery of the placenta
(B) B. The puerperium is a period of 42 days following delivery of the placenta
(C) C. The puerperium is a period of 21 days following delivery of the placenta
(D) D. The puerperium is a period of 14 days following delivery of the placenta

(61) General nursing care during post-partal period includes assessment of the:
(A) A. Breasts, abdomen, and fundus
(B) B. Breasts, abdomen, and musculoskeletal system
(C) C. Breasts, abdomen, and neurological responses
(D) D. Breasts, abdomen and congenital abnormalities

(62) At the end of labor, the cervix is:


(A) soft and flabby
(B) 2cm dilated
(C) 1cm dilated
(D) a slit

(63) You are taking vital signs observations on a postpartum client who delivered a
healthy newborn 4 hours ago. Her temperature is 100.2° F or 37.9 What is the priority
nursing action?
(A) A. Increase hydration by encouraging oral fluids.
(B) B. Document the findings.
(C) C. Retake the temperature in 15 minutes
(D) D. Notify the doctor
Postpartum fever is defined as a temperature of 38.7 degrees C (101.6 degrees F) or greater for
the first 24 hours or greater than 38.0 degrees C (100.4 degrees F) on any two of the first 10
days postpartum.
(64) .After delivery of a healthy newborn. Which time frame should the nurse relay to the
client regarding the return of bowel function?
(A) A. 7 days postpartum
(B) B. 3 days postpartum
(C) C. Within 2 weeks postpartum
(D) D. On the day of delivery

65) Nurse Dorothy is assessing a client for signs of postpartum depression. Which
observation, if noted in the new mother, would indicate the need for further assessment
related to this form of depression?
(A) A. The mother constantly complains of tiredness and fatigue.
(B) B. The mother is caring for the infant in a loving manner
(C) C. The mother demonstrates an interest in the surroundings.
(D) D. The mother looks forward to visits from the father of the newborn.

(66) Mrs.Nyamasege is a postnatal client who was diagnosed with superficial venous
thrombosis .In developing her care plan, the nurse anticipates that which intervention will
be prescribed?
(A) A. Ambulation eight to ten times daily
(​B) B. Administration of anticoagulants
(C) C. Elevation of the affected extremity
(D) D. Application of ice packs to the affected area

67) What is the best position to assess a postpartum woman’s fundus?


(A) A. On her right side.
(B) B. Semi-Fowler position.
(C) C. Supine with knees slightly flexed.
(D) D. In a chair, with feet elevated.

68) A new mother tells you that she is afraid of her baby and that she doubts she will be
able to love the baby. The most appropriate action would be to:
(A) A. Tell the client she will learn fast.
(B) B. Tell the client what a beautiful baby she has
(C) C. Encourage the client to talk about her feelings.
(D) D. Have a psychiatrist visit the client immediately

(69) The main function of oxytocin during puerperium is to stimulate:


(A) A. Estrogen production
(B) B. Progesterone production
(C) C. Contraction of uterine muscle and breast tissues
(D) D. Production of Human Placental Lactogen

(70) Fertilization occurs in one of the following parts of fallopian tube.


(A) A. Infundibulum
(B) B. Fimbrae
(C) C. Ampulla
(D) D. Isthmus

(71) In the female genital tract the sperms undergo physio-biochemical modification to
allow spermovum interaction. This modification is known as:
(A) A. Capacitation
(B) B. Transformation
(C) C. Reaction
(D) D. Activation

(72) The placenta originates from one of the following parts of the fertilized ovum
(A) A. Inner cell mass
(B) B. York sac
(C) C. Trophoblast
(D) D. Embryoblast

(73) The following structures in fetal circulation joins the umbilical cord to the inferior
vena cava
(A) A. Ductus arteriosus
(B) B. Foramen ovale
(C) C. Ductus venosus
(D) D. Hypogastric arteries

74) Antenatal care of HIV positive client includes:


(A) A. Antiretroviral therapy for life immediately HIV positive status is confirmed
(B) B. Sulfadoxine Pyrimethamine(SP)and cotrimoxazole prophylaxis daily until
delivery
(C) C. Antiretroviral therapy up to postnatal period from the time the CD4 count is
established
(D) D. Delay administration of Sulfadoxine Pyrimethamine until antiretroviral therapy is
started
Maternal and Newborn Health (MNH) Pillars
1. F​ amily planning and pre-pregnancy care​– To ensure that individuals and couples
have the
information and services to plan the timing, number and spacing of pregnancies.
2. F ​ ocused Antenatal Care​ – To prevent complications where possible and ensure that
complications
of pregnancy are detected early and treated appropriately.
3. E ​ ssential Obstetric Care ​– To ensure that essential care for the high-risk pregnancies
and
complications is made available to all women who need it.
4.​ Essential Newborn Care​ – To ensure that essential care is given to newborns from
the time they
are born up to 28 days in order to prevent complications that may arise after birth.
5. T ​ argeted Postpartum Care​– To prevent any complication occurring after childbirth
and ensure that
both mother and baby are healthy and there is no transmission of infection from mother
to child.
6.​ Post Abortion Care​ – to provide clinical treatment to all women and girls seeking
care, for
complications of incomplete abortion and miscarriage as well as counselling and
contraceptives.

(75) A client who is pregnant reports nausea and vomiting. The nurse reassures the client
that these symptoms will subside by;
(A) 6-9 WEEKS
(B) 10-13 WEEKS
(C) 14-17 WEEKS
(D) 18- 22 WEEKS

(76) The following type of exercises should be avoided in pregnancy


(A) A. Walking for 30 minutes
(B) B. Sexual intercourse
(C) C. Aerobic exercise
(D) D. Mountain climbing

(77) In class III cardiac disease in pregnancy, patient has:


(A) A. No limitation of physical activity, heart murmur on general examination
(B) B. Slight limitation of physical activity, no symptoms when at rest
(C) C. Marked limitation of physical activity, slight exertion leads to fatigue,
palpitations, dyspnea
(D) D. Inability to carry out any physical activity without discomfort, signs of cardiac
disease and heart failure even at rest

(78) The following statement is true concerning preeclampsia.


(A) A. Oedema is considered as part of the diagnosis of preeclampsia
(B) B. Preeclampsia is primarily hypertension during pregnancy
(C) C. The cardinal signs of preeclampsia include oedema and headache
(D) D. Preeclampsia is gestational hypertension plus proteinuria

79) Effects of gestational diabetes on the fetus include:


(A) A. Congenital malformation, macrosomia,
(B) Fetal hypoglycaemia, congenital anomalies
(C) C. Hypocalcemia, fetal hyperglyceamia
(D) D. Macrosomia, fetal hypoglycaemia

(80) Which of the following sex hormone is produced during throughout the menstrual
cycle
(A) A. Follicle stimulating hormone
(B) B. Luteinizing hormone
(C) C. Estrogen
(D) D. Progesterone

(81) The following hormones are produced by the pituitary gland


(​A) A. Follicle stimulating hormone and Luteinizing hormone
(B) B. Estrogen and Progesterone
(C) C. Follicle stimulating hormone and Estrogen
(D) D. Luteinizing hormone and Progesterone

82) Uterovesical pouch is formed when


(​A) A. Perimetriumis deflected over the bladder anteriorl​y
(B) B. Perimetriumis deflected over the rectum posteriorly
C. Endometrium is deflected over the bladder anteriorly
(D) D. Endometrium Is Deflected Over the rectum posteriorly

(83) The following hormone causes development of new blood vessels to the uterus
(A) A. Eostrogen
(B) Progesterone
(C) C. Luteinizing hormone
(D) D. Follicle stimulating hormone

84) Colostrum can be expressed at:-


(​A) A. 6 weeks gestation
(B) B. 12 weeks gestation
(C) C. late pregnancy
(D) D. 16weeks gestation

85) Management of nausea and vomiting among pregnant women include:-


(A) A. Skipping meals
(B) B. Eating small meals frequently
(C) C. Avoiding crackers before getting up in the morning
(D) D. Encouraging highly seasoned foods

(86) The following is caused by deficiency of zinc:-


(A) A. Pre-eclampsia
(B) B. Night blindness
(C) C. Impaired immune system
(D) D. Increased risk of spontaneous abortions

(87) Ptyalism refers to:


(A) A. Excessive nausea and vomiting in pregnancy
(B) B. Lack of appetite
(C) C. An abnormal craving for non-food substances of little or no nutritional value
(D) D. Increased salivation

(88) The normal level of haemoglobin (Hb) on a pregnant woman is:-


(A) 10-15g/dl
(B) 12-14g/dl
(C) 9-14g/dl
(D) 11-12g/dl

(89) The trophoblast cells of the developing zygote form the:-


(A) A. Fetus and amnion.
(B) B. Placenta and fetus.
(C) C. Chorion and amnion
(​D) D. Placenta and Chorion

92) In focused antenatal care, the second dose of sulfadoxine pyrimethamine(SP) is


administered at gestation of:
(A) 16-28 weeks
(B) 20-30 weeks
(C) 28-34 weeks
(D) 34-36 weeks

(93) The diastolic pressure is usually not affected by excitement, its increase therefore is
more significant in assessment of pregnant women for pre-eclampsia. The range of
change in mmHg is:
(A) 10-15
(B) 15-20
(C) 15-30
(D) 20-30

(94) The management of placenta praevia type three when the fetus has died in utero
includes:
(A) A. Preparation of women for emergency c-section
(B) B. Induction of labour by use of syntocinon
(C) C. Gentle palpation through vaginal fornices to determine whether posterior or
anterior
(D) D. Complete bed rest and speculum examination on the third day

(95) Fetal axis pressure is a physiological change that is more significant during:
(A) A. Early stages of labor transmitting the force of contraction to the upper pole of
fetus
(B) B. Second stage of labor when membranes are intact
(C) C. Active and second stages of labor when membranes are ruptured
(D) D. Second stage of labour transmitting the force of contraction to the lower pole of
the uterus

(96) If the membranes rupture at cervical dilatation of 8 cm, a vaginal examination


performed rule out:
(A) A. Obstructed labour, cord prolapsed, fetal compromise
(B) B. Cord prolapsed, cord presentation, fetal distress
(C) C. Obstructed labour, cord prolapsed, prolonged labour
(D) D. Maternal distress, prolonged labour, cord presentation

(97) At primary level of anti-retroviral prophylaxis for PMTCT( Prevention of mother to


child transmission), a HIV –positive woman should take:-
(A) A. Nevirapine 300 mgs at onset of labour
(B) B. Nevirapine 200 mgs at onset of labour
(C) C. Zidovudine 300mgs 3 hourly for 12 hours
(​D) D. Zidovudine 2mg/kg body weight within 3 days of delivery

(98) After birth, the functional closure of ductus arteriosus takes place within:
(A) 0-1 hour
(B) 2-4hour
(C) 4-6 hours
(D) 8-10 hours

(99) 50. The anti-opiod drug used in resuscitation of the newborn is


(A) A. Konakion 0.5 mg intramuscularly
(B) B. Dexamethasone1-2mg IV
(​C) C. Narcan 0.1mg/kg body IV
(D) D. Naloxone 0.5mg IV

​Exam 2

1. Duodenal ulcers,
(A) Are more common in people above 50 years
(B) Are more common in females than males
(C) Have a higher risk of malignancy than gastric ulcers
(​D) Pain is commonly experienced 3 hours after meals. 2. The following is a mineralocorticoid;

2. ​The following is a mineralocorticoid


(A) Cortisol
(B) Adrenal corticotropic hormone
(C) Aldosterone
(D) Epinephrine

3. ​Post intracranial surgery a patient requires strict temperature regulation mainly to;
(A) Detect infection
(B) Prevent increased metabolic demand
(C) Prevent hypothermia
(D) Enhance cerebral perfusion

4. ​Management of a patient during a seizure include;


(A) Restricting the jerking movements to prevent injury
(B) Inserting an oral airway to protect the tongue
(​C) Putting pillows and raising the side rails if the patient is in bed
(D) Removing pillows and raising the side rails if the patient is in bed.

5. ​A patient with a sprained ankle should be advised to avoid;


(A) Resting the foot
(B) Applying a heating pad
(C) Applying an elastic compression bandage
(D) Elevating the ankle on a pillow while sitting or lying down

6. Risk for osteoporosis is greatest in;


(A) A 36-year-old man who has asthma
(B) A 70-year-old man who consumes alcohol
(C) A sedentary 65-year-old woman who smokes cigarette
(D) A 25-year-old woman who jogs

7. Laboratory findings that are consistent with hyperthyroidism include


(A) Decreased serum triiodothyronine (T3) and thyroxine (T4) levels
(B) Elevated serum thyrotropin releasing hormone (TRH) levels
(C) Decreased radioactive iodine uptake
(D) Increased free triiodothyronine (T3) and thyroxine (T4) levels

8.​. A patient newly diagnosed with HIV/AIDS who laments, "I can't believe I got this virus.
I am going to die and everyone will know it was due to my behavior. How could I be so
stupid?" is in the stage of:
(A) Depression
(B) Denial
(C) Acceptance
(D) Bargaining
9. Substances that create colloid osmotic pressure in the capillaries include;
(A) Albumin, globulin
(B) Plasma, potassium
(C) Globulin, sodium
(D) Albumin, potassium

10. Discharge instructions for a client with chronic respiratory acidosis should include
(A) Discussing how to plan for periods of increased activity
(B) Teaching about a low protein, low carbohydrate diet
(C) Demonstrating exercises to increase vital capacity
(D) Encouraging participation in activities such as jogging

11. The predilection of HIV for certain cells in the immune system results in
(A) More T4 lymphocytes and general lymphocytopenia
(B) Fewer activated B-cells and hypogammaglobulinemia
(C) Increased response in cutaneous hypersensitivity
(D) Susceptibility to opportunistic infection and neoplasm

12. The most common preventable cause of valvular heart disease is:
(A) Congenital disease acquired intrauterine
(B) Calcium deposits and thrombus formation
(C) Marfans syndrome
(D) Beta-hemolytic streptococcal infection

13. An anorexic client states that her stomach is "fat" and that all the food she is being forced to
eat is making her obese. The nurse's best reply is:
(A) "Your body is starting to work normally again and is getting stronger and healthier
(B) "You have a bloated stomach because there is too much salt in your food"
(C) "No, it's not, you look much better now than when you were first admitted"
(D) "Maybe we have been feeding you too much and you're gaining weight too fast"

14. The major factors associated with obesity include:-


(A) Genetic, nutritional, level of activity.
(B) Nutritional, level of activity, socio-cultural.
(C) Nutritional, social, genetic.
(D) Level of activity, genetic, psychological.

15. Pelvic inflammatory diseases include:


(A) Salpingitis, cervicitis, candidiasis
(B) Cervicitis, endometritis, oophoritis.
(C) Trichomoniasis, salpingitis, urethritis.
(D) Candidiasis, toxic shock syndrome, endometritis.
16. Cholelithiasis is associated with:-
(A) Abnormal metabolism of cholesterol and bile salts.
(B) Abnormal metabolism of lipids and bile acids.
(C) Chronic pancreatitis.
(D) Hepatitis.

17. Chvostek’s sign is used for diagnosis of:-


(A) Hypercalcemia resulting from hyperparathyroidism.
(B) Hypercalcemia resulting from hyperpituitarism.
(C) Hypocalcemia resulting from hypoparathyroidism.
(D) Hypocalcemia resulting from hypothyroidism.

18. Glomerulonephritis usually follows:-


(A) Staphylococcal infection.
(B) Gonococcal infection.
(C) Gonococcal infection.
(D) Streptococcal infection

19. The teeth that commonly get impacted are the :-


(A) Molars.
(B) Incisors.
(C) Canine.
(D) Wisdom.

20. Body mechanics usually prescribed for clients with low pain include:-
(A) Bending at knee and hips, standing straight, carrying objects close to the body.
(B) Bending at the waist carrying objects close to the body, standing straight.
(C) Sleeping on the side carrying objects farther from the body, bending at the knees and hips.
(D) Bending at the waist, sleeping on the side, standing straight.

21. When a nurse is monitoring a patient for cerebrospinal fluid (CSF) leak he/she should
observe for:-
(A) Halo sign.
(B) Babinski reflex.
(C) Iliopsoas sign.
(D) Blumberg sign.

22. The commonest cause of toxic goitre (Grave’s disease) is: -


(A) Pituitary tumours
(B) Benign thyroid
(C) Lack of iodine in the diet
(D) Immunoglobulins reaction
23. Parathyroid hormone maintains fluid and electrolyte balance by
(A) Promoting absorption of calcium from the intestines
(B) Inhibiting release of calcium from the bone
(C) Increasing the excretion of phosphate ions by the kidneys
(D) Acting on the renal tubules to retain water and to decrease urinary output

24. The Na+ K+ ATPase pump: -


(A) Extrudes potassium ions from the cell and takes two sodium ions into the cell
(B) Extrudes three sodium ions from the cell and takes two potassium ions into the cell
(C) Has a coupling ratio of 2:1
(D) Is found mainly in the intravascular fluid compartment

25. Conductive hearing loss can be caused by: -


(A) Ototoxic medication, otitis externa
(B) Presbycusis, ototoxic medication
(C) Meniere’s disease, otosclerosis
(D) Otosclerosis, tympanic membrane retraction

26. During thoracentesis


(A) Fluid is aspirated from the peritoneal space
(B) Consent is not necessary
(​C) Medications can be instilled into the pleural space
(D) Local anaesthesia is not advisable

27. The carcinogenic risk factors for laryngeal cancers include:


(A) a. Tar products, mustard gas, chronic laryngitis
(B) b. Asbestos, chronic laryngitis, voice straining
(C) c. Voice straining, high fibre diet, tobacco
(D) d. Asbestos, tobacco, tar products

28. Sympathetic stimulation of the cardiovascular system causes: -


(A) a. Increased heart rate and constriction of coronary arteries
(B) b. Constriction of GIT secretory gland vessels with increased flow of digestive juices.
(C) c. Increased peripheral resistance and dilatation of coronary arteries
(D) d. Increased force of heart contraction & reduced peripheral resistance.

29. A deteriorating condition for a patient with head injury will be indicated by;
(A) Widening pulse pressure and irregular respiratory pattern.
(B) Narrowing pulse pressure and escalation of discomfort.
(C) Bradycardia and Kussmaul breathing
(D) Oliguria and narrow pulse pressure.

30. In Fallot’s tetralogy there is


(A) Pulmonary artery stenosis, ventricular septal defect, aortic misplacement & right ventricular
hypertrophy.
(B) Coarctation of the aorta, patent ductus arteriosus, aortic stenosis and ventricular septal
defect.
(C) Ventricular hypertrophy, coarctation of the aorta, atrial septal defect and Blue babies’
syndrome
(D) Atrial-ventricular hypertrophy, coarctation of the aorta, ventricular septal defect and
pulmonary artery stenosis.

31. An individual considered overweight has a basal metabolic rate (BMI) of:
(A) a. 21-25
(B) b. 25-29
(C) c. 30-39
(D) d. Above 40

32. The STI that particularly increases transmission and acquisition of HIV include:
(A) Gonorrhoea, chancroid
(B) Candidiasis, gonorrhoea
(C) Chancroid, Chlamydia
(D) Syphilis, genital herpes

33. The position an unconscious patient should be nursed in is:


(A) Recumbent
(B) Lithotomy
(C) Prone
(D) Semi-prone

34. Parklands’ formula is used to calculate


(A) Total burnt surface area
(B) Fluid replacement
(C) Thickness of the burn
(D) Hemo-concentration

35. The body’s internal environment which is tightly controlled and kept within normal limit is:
(A) Control feedback mechanism
(B) Compensatory mechanism
(C) Homeostasis
(D) Equilibrium

36. Before barium meal is performed, the nurse should:


(A) Position the patient in supine position
(B) Perform a head to toe examination
(​C) Take history and ask for history of allergic reactions to barium
(D) Alert the doctor if the patient had severe vomiting

37. The vascular coat of eye ball that lies between the sclera and retina is
(A) Uvea
(B) Iris
(C) Ciliary body
(D) Choroid

38. Following thyroidectomy, the nurse suspects damage or removal of parathyroid gland when
the patient develops:
(A) Muscle weakness and weight loss
(B) Hyperthermia and severe tachycardia
(C) Hypertension and difficulty in swallowing
(D) Laryngeal stridor and tingling in the hands and feet

39. increased concentration of red blood cells due to excessive production of erythropoietin is
referred to as:
(A) primary polycythemia
(B) leukopenia
(C) secondary polycythemia
(D) agranulocytosis

40. The following observations are noted on the patient on underwater seal drainage:
(A) a. Vital signs, patent connections, colour of drainage
(B) b. Vital signs, colour of drainage, amount of drainage
(C) c. Vital signs, patient understanding of the procedure, amount of drainage
(D) d. Vital signs, patient understanding of the procedure, cyanosis

41. Fractures of the base of the skull will manifest with


(A) Hemorrhage from the nose, the pharynx or the ears
(B) Cerebrospinal fluid leakage from the ears
(C) Bloody spinal fluid
(D) Persistent localized pain on swelling

42. Which cranial nerve is involved when the patient presents with facial pain, diminished
corneal reflex and chewing dysfunction:
(A) III
(B) IV
(C) V
(D) VI

43. The main long-term complication following radical mastectomy is


(A) Phantom pain
(B) Tingling and numbness
(C) Lymphoedema
(D) Paralysis

44. Reverse barrier nursing is aimed at:


(A) Nursing a patient in a corner bed with a sink
(B) Taking precautions to protect the patient
(C) Taking precautions to protect other patients
(D) Nursing the patient who has a highly infectious disease

45. Steroid nasal sprays are used in management of sinusitis to:


(A) Moisten secretions
(B) Prevent nasal crusting
(C) Reduce mucosal inflammation
(D) Thin the nasal secretions

46. Restoration of bone fragments to the original position following a fracture is called:
(​A) Reduction
(B) Immobilization
(C) Rehabilitation
(D) Restoration of function

47. Haemophilia A results from deficiency of factor:


(A) VII
(B) IX
(C) III
(D) V

48. The accumulation of thick, purulent fluid within the pleural space with fibrin development is
characteristic of:
(A) Empyema
(B) Effusion
(C) Pleurisy
(D) Emphysema

49. The Nursing council of Kenya (NCK) is established under the Nurses Act Cap.
(A) 274
(B) 247
(C) 257
(D) 254

50. The nursing diagnosis that best describes a patient with psoriasis who has quit her job as a
receptionist because she feels her appearance is disgusting to customers is:
(A) Ineffective coping related to loss of skin integrity
(B) Social isolation related to loss of skin integrity
(C) Impaired skin integrity related to presence of lesions
(D) Anxiety related to lack of knowledge of the disease process

51. The anterior posterior fontanelles normally closes at the age of:
(A) 6weeks and 18 weeks respectively
(B) 18 weeks and 6 weeks respectively
(C) 18 months and 6 weeks respectively
(D) 6 months and 18 months respectively

52. The trophoblast cells of the developing zygote from the:-


(A) Fetus and amnion
(B) Placenta and fetus
(C) Chorion and amnion
(D) Placenta and chorion

53. When performing Leopold's maneuver, the purpose of the first maneuver is to determine
(A) Fetal part lying in the fundus and presentation
(B) Location of fetal back and position
(C) Engagement of the presenting part
(D) The degree of flexion of fetal head

54. If Mrs. Cola’s menstrual period was on 27.7.08, her expected date of delivery will be:
(A) 3.4.09
(B) 4.5.09
(C) 3.5.09
(D) 4.4.09

55. The most important benefit of vestibular stimulation in the normal newborn is improvement
of
(A) Sucking capability
(B) Tactile sensitivity
(C) Motor coordination
(D) Attention span

56. In focused antenatal care, the second dose of sulfadoxine pyrimethamine (sp) is
administered at gestation of:-
(A) 16-28 weeks(First dose)
(B) 20-30 weeks.
(C) 28-34 weeks. (Second dose)
(D) 34-36 weeks.
Rationale:
Focused Antenatal Care – To prevent complications where possible and ensure that
complications of pregnancy are detected early and treated appropriately.
The objectives of focused antenatal care are:
● Early detection and treatment of problems
● Prevention of complications using safe, simple and cost-effective interventions
● Birth preparedness and complication readiness
● Health promotion using health messages and counseling
● Provision of care by a skilled attendant
It is recommended a pregnant mum attend a minimum of 4 visits.
● First visit less than 16 weeks
● Second visit 16 - 28 weeks
● Third visit 28-32 weeks
● Fourth visit 32 – 40 weeks
- If the first visit is after 16 weeks, give: In malaria endemic areas:
sulfadoxine/pyrimethamine (IPT), three tablets once to be taken at the facility
under supervision(DOT)
● The Ministry of Health Guidelines on Malaria directs us to give SP to pregnant
women in endemic malaria areas, at least twice during each pregnancy, even if
she has no physical signs and her haemoglobin is within normal range.
● Administer IPT with each scheduled visit after quickening (16 weeks) to ensure
women receive at least 2 doses at an interval of at least 4 weeks.
● IPT should be given under Directly Observed Therapy (DOT) in the ANC clinic and
can be given on an empty stomach

57. The diastolic pressure is usually not affected by excitement, its increase is therefore more
significant in assessment of pregnant women for pre-eclampsia. The range of change in mmHg
is:-
(A) 10-15
(B) 15-20
(C) 15-30
(D) 20-30
Rationale :
58. The management of placenta previa type three when the fetus has died in utero includes:-
(A) Preparation of woman for emergency caesarian section.
(B) Induction of labour by use of syntocinon.
(C) Gentle palpation through vaginal forncles to determine whether posterior or anterior.
(D) Complete bed rest and speculum examination on third day.

59.The hormonal factor responsible for uterine contractions at onset of labour are:-
(A) Increase release of oxytocin progesterone from deciduas at term.
(B) Formation of oxytocin and estrogen in the fetal circulation
(C) Rise of oxytocin and estrogen in the fetal circulation.
(D) Optimum levels of estrogen and rise of prostaglandins in maternal circulation.
60. Fetal axis pressure is a physical change that is more significant durin
(A) Early stages of labour transmitting the force of contraction to the upper pole of fetus.
(B) Second stage of labour when membranes are intact.
(C) Active and second stages of labour when membranes are ruptured.
(D) Second stage of labour transmitting the force of contraction to the lower pole of the uterus.

61. If membranes rupture at cervical dilation of 8 centimeters, a vaginal examination performed


should rule out:-
(A) Obstructed labour, cord prolapse, fetal compromise.
(B) Cord prolapse, cord presentation, fetal distress.
(C) Obstructed labour, cord prolapse, prolonged labour.
(D) Maternal distress, prolonged labour, cord presentation.

62. The partograph is an effective tool in management of women in labour, a good progress in
the active phase will be:-
(A) Cervical dilatation plotted progressively on alert line within 7 hours.
(B) Uterine contractions increase in strength from 20-25 seconds within 8 hours.
(C) Cervical dilation plotted on the action line and progressive descent.
(D) Cervical dilatation plotted the right of the action line.

63. At primary level of ant-retroviral prophylaxis for PMTCT (prevention of mother to child
transmission), a HIV-positive woman should take:-
(A) Nevirapine 300 mgs at onset of labour
(B) Nevirapine 200 mgs at onset of labour
(C) Zidovudine 300 mgs 3 hourly for 12 hours.
(D) Zidovudine 2mg/ kilogram body weight within three days of delivery

64. The initial characteristics of rigid cervix in cervical dystocia include:-


(​A) Thin, tight, unyielding.
(B) Thick, tight, edematous.
(C) Long, thick, edematous
(D) Thick, edematous, unyielding

65. Perineal trauma is common in delivery of face to pubis due to:-


(A) Irregular shape of the face as a presenting part.
(B) Persistent occipito-posterior position when occiput becomes the leading part.
(C) Large presenting mentovertical diameter
(D) Large presenting occipito-frontal and biparietal diameters.

66. After birth, the functional closure of ductus arteriosus takes place within:-
(A) 1-2hours.
(B) 2-4 hours
(C) 4-6 hours
(D) 8-15 hours

Rationale: Normally, functional closure of the ductus arteriosus occurs by about 15


hours of life in healthy infants born at term. This occurs by abrupt contraction of the
muscular wall of the ductus arteriosus, which is associated with increases in the partial
pressure of oxygen (PO2) coincident with the first breath

67. The anti-opioid drug used in resuscitation of the newborn is:-


(A) Konakion 0.5 mg intramuscularly.
(B) Dexamethasone 1-2mg Intravenously.
(C) Narcan 0.1 mg/kilogram body weight intravenously
(D) Naloxone 0.5 mg intravenously.

68. Injury to the fetal scalp due to pressure by girdle of contact is known as:-
(A) Cephalhematoma.
(B) Pericranial hematoma.
(C) Subdural oedema.
(D) Caput succedaneum.

Rationale :
CAPUT SUCCEDANUM (CAPUT) : Caput succedaneum is a diffuse swelling of the scalp
in a newborn caused by pressure from the uterus or vaginal wall during a head-first
(vertex) delivery. It is caused by the mechanical trauma of the initial portion of scalp
pushing through a narrowed cervix. The swelling may be on any portion of the scalp,
may cross the midline (as opposed to a cephalohematoma), and may be discoloured
because of slight bleeding in the area. Moulding of the head is common in association
with a caput succedaneum. Physical examination confirms that the swelling is a caput
succedaneum. No testing is necessary. No treatment is necessary, and it usually heals
spontaneously within a few days. Complete recovery can be expected, with the scalp
regaining its normal contour. Possible Complications include Jaundice which is a result
of the bruise breaking down into bilirubin.
Prevention:
A caput succedaneum is more likely to form during a prolonged or difficult delivery. This
is especially true after the membranes have ruptured, thus removing the protective
cushion of the amniotic sac. Vacuum extraction can also increase the chances of a caput
succedaneum. However, a caput succedaneum is sometimes identified by prenatal
ultrasound even before labour or delivery begins. It has been found as early as 31 weeks
of gestation. More often than not, this is associated with either premature rupture of the
membranes or too little amniotic fluid (oligohydramnios). All other things being equal,
the longer the membranes are intact, the less likely a caput is to form. Nevertheless, a
caput succedaneum can form before or during birth even in the absence of any
identifiable risk factor. Good prenatal care and management of labour and delivery can
reduce the chances of this minor problem

69. The peak action syntocinon administered to prevent postpartum hemorrhage is felt in:-
(A) 2 ½ minutes
(B) 2 minutes
(C) 2 seconds
(D) 2 ½ seconds

70. ​The exogenous causative organisms to puerperal infection include:-


(A) Escherichia coli, clostridium welchii.
(B) Streptococcus faecalis, haemolytic streptococcus
(C) Staphylococcus aureus, Escherichia coli.
(D) Haemolytic streptococcus aureus

71. Mrs Dodo para 6+ 0 comes to the postnatal clinic at six weeks and demands for
bilateral tubal ligation, the midwife should:-
(A) Reassure and encourage her to continue with lactational amenorrhea method.
(B) Schedule for return date in order to prepare for the procedure.
(C) Check whether the pregnancy test is negative then provide the demanded method
accordingly.
(​D) Take in-depth medico-social history and encourage partner participation.

72. Fetal causes of early pregnancy loss include:-


(A) Structural abnormalities of the genital tract
(B) Bicornulate uterus and fibroids
(C) Chromosomal abnormalities of the conceptus
(D) Infections such as rubella, listeria and Chlamydia

73. The outcome of labor is not dependent on:-


(A) Effectiveness of uterine contractions
(B) Maternal size
(C) The “give” of the pelvic joints
(D) The degree of moulding of the fetal head

74. In monitoring fetal well being using the fetal kick chart, you will tell the client to
notify you if she counts:-
(A) More than 10 movements in 3hours
(B) Less than 10 movements in 3hours
(C) More than 10 movements per hour
(D) Less than 10 movements per hour

75. During discharge, an important advice for the breastfeeding mother is to:-
(A) Routinely avoid onions, cabbage and chocolate
(B) Keep her calcium intake the same as during pregnancy
(C) Increase her calories over her pregnancy requirements
(D) Return to her pre-pregnancy intake for proteins but not calories

76. The part of the uterus in which a contraction begins is:-


(A) Cornua
(B) Fundus
(C) Isthmus
(D) The body or corpus

77. The follicle stimulating hormone and luteinizing hormone are secreted by the:-
(A) Ovaries and testes
(B) Posterior pituitary
(C) Anterior pituitary gland
(D) Hypothalamus

78. Uterine contractions are controlled by the:-


(A) Central nervous system
(B) Sympathetic nervous system
(C) Peripheral nervous system
(​D) Autonomic nervous system

79. The volume of amniotic fluid at 40 weeks gestation is:-


(A) 500 to 1000 ml
(B) 350 to 500 ml
(​C) 500 to 800 ml
(D) 1000 to 1500 ml

80. The ectoderm differentiates to form the:-


(A) Cardiovascular system
(B) Peripheral nervous system
(C) Respiratory tract
(D) Connective tissues
81. The hormone that maintains endometrium and stimulates maternal metabolism and
development of breast alveoli during pregnancy is:-
(A) Human chorionic gonadotropin
(B) Human placental lactogen
(C) Oestrogen
(D) Progesterone

82. The anteroposterior diameter of the pelvic outlet measures:-


(A) 12CM
(B) 13CM
(C) 11CM
(D) 10CM

83. During pregnancy, the glomerular filtration rate


(A) Only changes when pathological conditions are present
(B) Is directly influenced by the woman’s posture
(C) Increases significantly above pre-pregnant levels
(D) Makes the most dramatic changes near birth

84. Signs of cardiac decompensation in a pregnant woman with cardiac disease are
likely to appear at
(A) 28-32 weeks gestation
(B) 36-40 weeks gestation
(C) 12-16 weeks gestation
(D) 20-24 weeks gestation

85. The mechanism of labor that allows the fetal head to present itself to fit the widest
anteroposterior diameter of the pelvic cavity is:-
(A) Flexion
(B) Internal rotation
(C) Descent
(D) Extension

86. Closure of the newborns foramen ovale occurs when:-


(A) Blood flows from the pulmonary artery to the aorta
(B) PO2 is increased causing constriction to occur
(C) The umbilical cord is severed
(D) Left atrial pressure exceeds right atrial pressure
87. The newborn with post maturity syndrome is at high risk for cold stress due to:-
(A) Absence of vernix
(B) Decreased subcutaneous fat
(C) Parchment like skin
(D) Extended posture

88.Presence of Meconium in the newborn’s lungs:-


(A) Leads to respiratory alkalosis
(B) Prevents air leaks
(C) Traps inspired air in the alveoli
(D) Prevents air from entering the airway

89. Symptoms of superficial thrombophlebitis include:-


(A) Local redness and tenderness in a portion of the vein
(B) Edema of the ankle and lower leg
(C) Pain in the affected leg and foot
(D) Positive Homan’s sign

90. Direct causes of placenta abruption include:-


(A) Abnormally short umbilical cord, sudden loss of uterine volume
(B) Defective vascularization of the deciduas, previous uterine surgery
(C) Injury to the abdomen from fall or accident, multiple pregnancy
(D) Advanced maternal age, injury to the abdomen from fall or accident

91. Persistence of moro reflex beyond the age of 6months is usually suggestive of:
(A) Immaturity
(B) Brain damage
(C) Mental retardation
(D) Brachial plexus palsy

92. Babies born with intrauterine growth retardation are usually prone to;
(A) Congenital abnormalities, hyperglycemia
(B) Neonatal hyperglcemia, asphyxia
(C) Neonatal asphyxia, birth defects
(D) Neonatal tetanus, asphyxia

93. Mrs. Ug, presents in the maternity ward with pulsating prolapsed umbilical cord. The
recommended nursing position would be;
(​A) Knee-chest position, exaggerated Sim's position
(B) Fowler's position, McRoberts manoeuvre position
(C) Sim's position, trendelenburg position
(D) Prone position, knee-chest position

94.Lochia serosa is composed of:


(A) Decidua, vernix, leucocytes
(B) Leucocytes, cervical mucus, decidua
(C) Cervical mucus, cholestrin crystals, lanugo
(D) Leucocytes, cervical mucus, erythrocytes

95. During second stage of labour, the diameter that distends the vaginal orifice in face
presentation is:
(A) Occipito frontal
(B) Submento-vertical
(C) Submento-bregmatic
(D) Mentovertical

96. The right time for diagnosing HIV infection using antibody tests among children
born to HIV positive mother is at or after;
(A) 12 months of age
(B) 16 months off age
(C) 18 months of age
(D) D. 6 months of age

97. During management of severe eclampsia


(A) Anticonvulsant therapy should be given once daily
(B) Ketamine is the drug of choice in performing a caesarean delivery
(​C) Nifedipine can be used to lower the blood pressure
(D) Blood pressure should be monitored 6hourly

98. Benefits of breastfeeding to the infant include:


(A) Increased cognitive development, less chances of developing diabetes type 2, less
risk of extreme obesity
(B) Lowered risk of infection, less likely to become overweight, protection against
malignancies
(C) Strengthens bonding, protection against malignancies, decreased risk of
cardiovascular diseases
(D) Reduced risk of metabolic syndrome, lowered risk of developing necrotizing
enterocolitis, protection against diabetes type 1

99. The common age of onset for gonococcal ophthalmia neonatorum is:
(A) Between 21-28 days after birth
(B) Between 0-7 days after birth
(C) Between 7-14 days after birth
(D) At 28 days of age

100. Perinatal mortality refers to fetal and infants deaths including still births occurring
from 28 weeks of gestation to within the first:
(A) One week of life per 1000 live births
(B) Two weeks of life per 1000 live births
(C) Four weeks of life per 1000 live births
(D) Six weeks of life per 1000 live births

Type 3

1. Which of the following respiratory patterns indicate increased intracranial pressure in


the brain stem?
(A) Slow, irregular respirations
(B) Rapid, shallow respirations’
C) Asymmetric chest excursion
(D) Nasal flaring
Rationale:
The rising systolic pressure results in widened pulse pressures, bradycardia and
irregular breathing. As intracranial pressure continues to increase, the patient's heart
rate will increase, breathing will become shallow, periods of apnea will occur, and
blood pressure will begin to fall. Eventually an agonal rhythm will develop as
herniation begins, followed soon by cessation of brain stem activity, respiratory arrest
and cardiac arrest.

(2) A client has signs of increased intracranial pressure (ICP). Which of the following is
an early indicator of deterioration in the client’s condition?
(A) Widening pulse pressure
(B) Decrease in pulse rate
(C) Dilated fixed pupils
(D) Decrease in level of consciousness
Rationale :
Early signs of rising intracranial pressure include ​decreased level of consciousness,
restlessness, irritability and confusion.​ With a continued increase, speech, voluntary
movements, sensations and extraocular movements will slow. Additionally, T-wave
elevation will develop on the electrocardiogram. Your patient may only speak when
stimulated, have no voluntary movements and only respond to painful stimuli. As
pressure increases near the medulla, the patient may experience projectile vomiting
with no associated nausea, and cardiac arrhythmias can range from supraventricular
tachycardia to severe bradycardia. As coma develops, reaction to painful stimuli will
become reflexive and may disappear completely. When herniation of the brain is
imminent, loss of extraocular movement will occur, with the pupils dilating, becoming
unreactive and turning outward.

(3) The nurse administers mannitol to the client with increased intracranial pressure.
Which parameter needs close monitoring
(A) Muscle relaxation
(B) Intake and output
(C) Widening of the pulse pressure
(D) Pupil dilation
Rationale:
Mannitol (Osmitrol) is an osmotic diuretic used to treat cerebral edema (increased
intracranial pressure) and acute glaucoma. When administered, mannitol causes an
increase in plasma oncotic pressure (similar to excess glucose) that draws free water
from the extravascular space into the intravascular space, creating a volume
expansion. This fluid, along with the drug, is excreted through the kidneys, thereby
reducing cerebral edema and intracranial pressure.
However, if a ​higher dose of mannitol is given or it accumulates (as in kidney disease),
fluid overload that may cause life-threatening pulmonary edema​. An early sensitive
indicator of fluid overload is the new onset of crackles auscultated in the lungs. T​o
prevent these complications, clients require frequent monitoring of serum osmolarity,
input and output, serum electrolytes, and kidney function.

(4) Which activity should the nurse encourage the client to avoid when there is a risk for
increased intracranial pressure (ICP)?
(A) Deep breathing
(B) Coughing
(C) Turning
(D) Passive range of motion (ROM) exercises
Rationale
Intracranial pressure is the force exerted on the inside of the skull by the brain,
cerebral spinal fluid and blood. An increase in pressure caused by one component
necessitates compensatory responses by the others to maintain intracranial pressure
within the normal range of 0–15 mmHg. Intracranial pressure adjusts continuously
with the daily activities of living. ​Breathing, lifting and coughing cause increases in
intracranial pressure. ​The body is capable of autoregulating intracranial pressure
increases that stay below 35 mmHg, with corresponding systolic blood pressures
between 60–160 mmHg and cerebral perfusion pressures between 50–150 mmHg.

(5) For a client who is experiencing expressive aphasia, which nursing intervention is
most helpful in promoting communication?
(A) Speaking loudly
(B) Using a picture board
(C) Writing directions so client can read them
(D) Speaking in short sentence
Rationale
Broca's Aphasia (expressive)
Injury to the frontal regions of the left hemisphere impacts how words are strung
together to form complete sentences. This can lead to Broca’s Aphasia, which is
characterized by:

● Difficulty forming complete sentences.


● Leaving out words like “is” or “the.”
● Saying something that doesn’t resemble a sentence.
● Trouble understanding complex sentences.
● Making mistakes in following directions like “left, right, under and after.”
● Using a word that’s close to what you intend, but not the exact word; for
example, saying “car” when you mean “truck.”
● (6) The nurse is assessing the motor and sensory function of an unconscious
client. The nurse should use which technique to test the client’s peripheral
response to pain?

(A) Sternal rub


(B) Nail bed pressure
(C) Pressure on the orbital rim - (localizing pain)
(D) Squeezing of the sternocleidomastoid muscle
(7) The nurse is evaluating the neurological signs of a client in spinal shock following
spinal cord injury. Which observation indicates that spinal shock persists?
(A) Hyperreflexia
(B) Positive reflexes
(C) Flaccid paralysis
(D) D. Reflex emptying of the bladder

(8) The nurse has instructed the family of a client with stroke (brain attack) who has
homonymous hemianopsia about measures to help the client overcome the deficit.
Which statement suggests that the family understands the measures to use when
caring for the client?
(A) “We need to discourage him from wearing eyeglasses.”
(B) “We need to place objects in his impaired field of vision.”
(C) “We need to approach him from the impaired field of vision.”
(D) “We need to remind him to turn his head to scan the lost visual field.”

(9) The nurse is teaching a client with myasthenia gravis about the prevention of
myasthenic and cholinergic crisis. Which client activity suggests that teaching is most
effective?
(A) Taking medications as scheduled
(B) Eating large, well-balanced meals
(C) Doing muscle-strengthening exercises
(D) Doing all chores early in the day while less fatigue

(10) The nurse has given suggestions to a client with trigeminal neuralgia about
strategies to minimize episodes of pain. The nurse determines that the client needs
further teaching if the client makes which statement?
(A) “I will wash my face with cotton pads.”
(B) “I’ll have to start chewing on my unaffected side.”
(C) “I should not rinse my mouth if toothbrushing is painful.”
(​D) “I should rinse my mouth if toothbrushing is painful.”
Rationale
Trigeminal neuralgia (tic douloureux) presents with paroxysms of unilateral
excruciating facial pain along the distribution of the trigeminal nerve (CN V) that are
often triggered by touch, talking, or hot/cold air or intake.​ Carbamazepine (Tegretol)
is the drug of choice; the condition is not life-threatening

(11) The client is admitted to the hospital with a diagnosis of Guillain-Barre syndrome.
Which past medical history finding makes the client most at risk for this disease?
(A) Meningitis or encephalitis during the last 5 years
(B) Seizures or trauma to the brain within the last year
(C) Back injury or trauma to the spinal cord during the last 2 years
(D) Respiratory or gastrointestinal infection during the previous month

(12) A client has a neurological deficit involving the limbic system. On assessment,
which finding is specific to this type of deficit?
(A) Is disoriented to person, place, and time
(B) Affect is flat, with periods of emotional lability
(C) Cannot recall what was eaten for breakfast today
(D) Demonstrates inability to add and subtract; does not know who is the president of
the United States
Rationale:
The limbic system is a set of structures in the brain that deal with emotions and
memory. It regulates autonomic or endocrine function in response to emotional stimuli
and also is involved in reinforcing behavior .

(13) The nurse is evaluating the status of a client who had a craniotomy 3 days ago.
Which assessment finding would indicate that the client is developing meningitis as a
complication of surgery?
(A) A negative Kernig’s sign
(B) Absence of nuchal rigidity
(C) A positive Brudzinski’s sign
(D) A Glasgow Coma Scale score of 15

Kernig sign: One of the physically demonstrable symptoms of meningitis is Kernig's


sign. ​Severe stiffness of the hamstrings causes an inability to straighten the leg when
the hip is flexed to 90 degrees.
● Kernig's is performed by having the supine patient, with hips and knees flexed,
extend the leg passively. The test is positive if the leg extension causes pain.
● The Brudzinski's sign is positive when passive forward flexion of the neck
causes the patient to involuntarily raise his knees or hips in flexion.

(14) The nurse is admitting a client with Guillain-Barre syndrome to the nursing unit. The
client has ascending paralysis to the level of the waist. Knowing the complications of
the disorder, the nurse should bring which most essential items into the client’s room?
(A) Nebulizer and pulse oximeter
(B) Blood pressure cuff and flashlight
(C) Flashlight and incentive spirometer
(D) Electrocardiographic monitoring electrodes and intubation tray

(15) A client with myasthenia gravis has become increasingly weaker. The health care
provider prepares to identify whether the client is reacting to an overdose of the
medication (cholinergic crisis) or an increasing severity of the disease (myasthenic
crisis). An injection of edrophonium is administered. Which finding would indicate that
the client is in a cholinergic crisis?
(A) No change in the condition
(B) Complaints of muscle spasms
(C) An improvement of the weakness
(D) A temporary worsening of the condition
Rationale :
Myasthenia gravis is an autoimmune disease of the neuromuscular junction
resulting in fluctuating muscle weakness. Autoantibodies are formed against the
acetylcholine receptors, so fewer receptors are available for acetylcholine to bind. It is
treated with pyridostigmine (Mestinon), which increases the amount of acetylcholine
at the synaptic junction, augmenting neuromuscular signals and improving muscle
strength.
Infection, undermedication, and stress can precipitate a life-threatening
myasthenic crisis, which is characterized by oropharyngeal and respiratory muscle
weakness and respiratory failure. This client's infection and increasing difficulty
swallowing indicate the need for immediate intervention.
Side effects of pyridostigmine: nausea, diarrhea, vomiting , decreased pupil
dilation, increased urine output,watery eyes. Bad side effects: SOB, bradycardia,
headache, dizziness, new muscle twitching/cramps/tingling
Diagnosis of MG is done with tensilon test with edrophonium an
acetylcholinerase inhibitor,​ ​which is like an enzyme blockade of acetylcholinerase and
causes an increase of acetylcholine, meaning we have more cholinergic response.
During the test, the antidote given is atropine which is anticholinergic.

(16) A male client is having a lumbar puncture performed. The nurse would plan to
place the client in which position?
(A) Side lying with pillow under the hip
(B) Prone, with a pillow under the abdomen
(C) Prone , in slight Trendelenburg position
(​D) Side-lying with legs pulled up and bent down onto chest

(17) The nurse is positioning the female client with increased intracranial pressure.
Which of the following positions would the nurse avoid?
(A) Head midline
(B) Head turned to the side
(C) Neck in neutral position
(D) Head of bed elevated 30 to 45 degrees
Rationale:
Methods to prevent or reduce the rate of a rising ICP include elevating the head 30–45
degrees, keeping the neck in a neutral position, maintaining normal oxygen and carbon
dioxide levels, avoiding overhydration, avoiding the clustering of treatments,
preoxygenation prior to any suctioning, avoiding valsalva’s maneuvers, maintaining
normal body temperature, avoiding noxious stimuli and administering appropriate
medications.

(18) 18. The nurse is caring for the male client who begins to experience seizure activity
while in bed. Which of the following actions by the nurse would be contraindicated?
(A) Loosening restrictive clothing
(B) Restraining the client’s limbs
(C) Removing the pillow and raising padded side rails
(D) Positioning the client to side, if possible , with the head flexed forward
Rationale
Here are things you can do to help Never do any of the following things
someone who is having this type of
seizure: ● Do not hold the person down or try
to stop his or her movements
● Ease the person to the floor (if
outside).
● Turn the person gently onto one
side. This will help the person ● Do not put anything in the
person’s mouth. This can injure
breathe. teeth or the jaw. A person having a
● Clear the area around the person seizure cannot swallow his or her
of anything hard or sharp. This tongue
can prevent injury.
● Put something soft and flat, like a
folded jacket, under his or her
head(If outside) ● Do not try to give mouth-to-mouth
breaths (like CPR). People usually
● Remove eyeglasses.
start breathing again on their own
● Removing the pillow and raising
after a seizure
padded side rails
● Loosen ties or anything around the
neck that may make it hard to
breathe. Loosening restrictive ● Do not offer the person water or
clothing food until he or she is fully alert.
● Time the seizure. Find help if the
seizure lasts longer than 5
minutes.

(19) The nurse is assigned to care for a female client with complete right-sided
hemiparesis. The nurse plans care knowing that this condition:
(A) the client has complete bilateral paralysis of arms and legs
(​B) the client has weakness on the right side of the body including the face and tongue
(C) the client has lost the ability to move the right arm but is able walk independently
(D) the client has lost the ability to move the left arm but is able to walk independently

(20) A female client has experienced an episode of myasthenic crisis. The nurse would
assess whether the client has precipitating factors such as:
(A) getting too little exercise
(B) taking excess medication - (will cause cholinergic crisis instead)
(C) omitting doses of medication
(D) increasing intake of fatty foods

(21) A male client has an impairment of cranial nerve II. Specific to this impairment, the
nurse would plan to do which of the following to ensure clients to ensure client safety?
(A) Speak loudly to the client
(B) Test the temperature of the shower water
(C) Check the temperature of the food on the delivery tray
(D) Provide a clear path for ambulation without obstacles
Rationale
Cranial Nerves
S​ I -​ Olfactory​ smell
S​ II - ​Optic​ - vision (Snellen chart, what letters can you read. 20/40
indicates that you can see at 20ft what the “normal” eye should see at 40ft).

M​ ​III - ​Oculomotor​ - upward, outward, medial, lens accommodation


M​ IV - ​Trochlea​r -downward and in
M​ VI - ​Abducens​ - adducting eyes from side to side*

SM​ V - ​Trigeminal- ​facial sensation​, ​somatosensation​ on tongue, chew


SM​ VII - ​ Facial​ taste ⅔ of anterior tongue, facial expression

S​ VIII -​ Acoustic​- hearing and equilibrium (Romberg test - checks for


balance)

SM​ IX - ​Glossopharyngeal​ - ⅓ of posterior tongue taste, speech


SM​ X - Vagus - digestion, defecation and bradycardia

M​ XI - Shoulder shrug, spinal accessory innervates trapezius muscles


M​ XII - ​Hypoglossal​ - Tongue movement

(22) A male client is having a tonic-clonic seizures. What should the nurse do first?
(A) Elevate the head of the bed
(B) Restrain the client’s arms and legs
(C) Place a tongue blade in the client’s mouth
(D) Take measures to prevent injury
Rationale

(23) The nurse is teaching the female client with myasthenia gravis about the
prevention of myasthenic and cholinergic crises. The nurse tells the client that this is
most effectively done by:
(A) Eating large, well-balanced meals
(B) Doing muscle -strengthening exercises
(C) Doing all chores early in the day while less fatigued
(D) Taking medications on time to maintain therapeutic blood levels
(24) Which of the following neurotransmitters is known to be associated with sleep,
mood and appetite?
(A) A. Acetylcholine
(B) B. Dopamine
(C) C. Serotonin
(D) D. GABA

(25) 25. The neurons of the motor pathway control:


(A) A. Stress reaction
(B) B. Organs and glands
(C) C. Involuntary muscles
(D) D. Voluntary muscles
Rationale
Parkinson’s Disease - affects how you walk

(26) 26. The area of the brain which is involved in the understanding of meaning is:
(A) A. Broca’s area
(B) B. association areas
(C) C. Wernicke area
(D) D. Motor speech area
Rationale:
Broca's is for production and Wernicke's is for comprehension of language​. They are
cortical areas specialized for human language. Broca's area is found in the left inferior
frontal gyrus and Wernicke's area is located in the left posterior superior temporal
gyrus.
(27) 27. Parkinson’s disease , is caused by the degeneration of neurons in an area of the
brain called the:
(A) A. Substantia nigra
(B) B. Basal ganglia
(C) C. Cerebellum
(D) D. Corpus callosum

(28) 28. The division of the nervous system which controls the involuntary muscles,
organs, and glands is:
(A) A. Somatic nervous system
(B) B. Autonomic nervous system
(C) C. Central nervous system
(D) D. The motor pathway of the CNS
(29) 29. Parasympathetic system is also called
(A) A. Fight or flight system
(​B) B. Eat-drink and rest system
(C) C. Autonomic nervous system
(D) D. Somatic nervous system

(30) 30. A condition, causing the affected person to be unable to understand or


comprehend the language ( poor comprehension) with intact repetition (fluent output) is
termed as:
(A) A. Broca’s aphasia
(B) B. Global aphasia
(​C) C. Wernicke’s aphasia
(D) D. Conduction aphasia

(31) A client is unconscious following a tonic-clonic seizure. What should the nurse do
FIRST?
(A) A. Place the client in a side-lying position
(B) B. Check the pulse
(C) C. Administer Valium
(D) D. Place a tongue blade in the mouth (​ Never do this! Places client in danger!)

(32) 32. The nurse enters the room as the client is beginning to have a tonic-clonic
seizure. What action should the nurse implement first?
(A) A. Determine if the client is incontinent of urine or stool.
(B) B. Provide the client with privacy during the seizure.
(C) C. Note the first thing the client does in the seizure.
(D) D. Assess the size of the client’s pupils.

(33) 33. The nurse is teaching a class on the prevention of cerebrovascular accidents.
Which of the following risk factors should the nurse identify as the most important
factor contributing to a stroke?
(A) A. Sedentary lifestyle
(B) B. Smoking
(C) C. Obesity
(D) D. Hypertension

(34) 35. The nurse is assessing a 37-year-old client diagnosed with multiple sclerosis.
Which of the following symptoms would the nurse expect to find?
(A) A. Flaccid muscles
(B) B. Tremors at rest
(C) C. Absent deep tendon reflexes
(D) D. Vision changes
Rationale
Multiple sclerosis (MS) is a condition that can affect the brain and spinal cord, causing a
wide range of potential symptoms, including problems with vision, arm or leg movement,
sensation or balance. It's a lifelong condition that can sometimes cause serious
disability, although it can occasionally be mild.In many cases, it's possible to treat
symptoms. Average life expectancy is slightly reduced for people with MS.It's most
commonly diagnosed in people in their 20s and 30s, although it can develop at any age.
It's about 2 to 3 times more common in women than men.MS is 1 of the most common
causes of disability in younger adults.​Some of the most common symptoms of Multiple
Sclerosis include:

❏ fatigue
❏ vision problems
❏ numbness and tingling
❏ muscle spasms, stiffness and weakness
❏ mobility problems
❏ pain
❏ problems with thinking, learning and planning
❏ depression and anxiety
❏ sexual problems
❏ bladder problems
❏ bowel problems
❏ speech and swallowing difficulties

(35) 36. If a male client experienced a cerebrovascular accident (CVA) that damaged
the hypothalamus, the nurse would anticipate that the client has problems with:
(A) A. Thinking and reasoning
(B) B. Body temperature control
(C) C. Balance and equilibrium
(D) D. Visual acuity

(36) 37. During recovery from a cerebrovascular accident (CVA), a female client is given
nothing by mouth, to help prevent aspiration. To determine when the client is ready for a
liquid diet, the nurse assesses the client’s swallowing ability once each shift. This
assessment evaluates
(A) A. Cranial nerves III and V
(B) B. Cranial nerves IX and X
(C) C. Cranial nerves I and II
(D) D. Cranial nerves VI and VIII
Rationale:
SM​ IX - ​Glossopharyngeal​ - ⅓ of posterior tongue taste, speech
SM​ X - Vagus - digestion, defecation and bradycardia
The c ​ ranial nerves​ associated with the swallowing process are the t​ rigeminal​ (V), f​ acial
(VII​), g
​ lossopharyngeal​ (​IX​), v
​ agus​ (X), accessory (​XI​) - usually not considered - and
hypoglossal​ (​XII​).

(37) After striking his head on a tree while falling from a ladder, a young man age 18 is
admitted to the emergency department. He’s unconscious and his pupils are
nonreactive. Which intervention would be the most dangerous for the client?
(​A) A. Perform lumbar puncture
(B) B. Elevate the head of his bed
(C) C. Give him a barbiturate
(D) D. Place him on mechanical ventilation

(38) A male client is admitted with a cervical spine injury sustained during a diving
accident. When planning this client’s care, the nurse should assign highest priority to
which nursing diagnosis?
(A) A. Disturbed sensory perception (tactile)
(B) B. Self-care deficit: dressing /grooming
(C) C. Impaired physical mobility
(D) D. Ineffective breathing pattern

(39) Demyelination is defined as;


(A) A. Loss of an axon in a neuron
(B) B. Loss of dendrites in a neuron
(C) C. Loss of protective sheath surrounding neurons
(D) D. Loss of connective tissue between neurons

(40) Failure of muscle coordination , including unsteady movements and staggering


walk, due to disorders in the cerebellum is called?
(A) A. Anoxia
(B) B. Dyslexia
(C) C. Paraplegia
(D) D. Ataxia
(41) Which part of the brain regulates heartbeat, breathing and other vital functions?
(A) A. Grey matter
(B) B. Brain stem
(C) C. Occipital lobe
(D) D. White matter

(42) Common complications of spinal cord injury include each of the following
EXCEPT?
(A) A. Pressure sores
(B) B. Deep vein thrombosis
(C) C. Diarrhoea
(D) D. Urinary Tract infections

(43) 45. Broca’s area is located in the


(A) A. Brain stem
(B) B. Frontal lobe
(C) C. Parietal lobe
(D) D. Temporal lobe

(44) Wernicke area is located in the


(A) A. Brain stem
(B) B. Parietal lobe
(C) C. Temporal lobe
(D) D. Frontal lobe
Rationale
Broca's area is found in the left inferior frontal gyrus and Wernicke's area is located in
the left posterior superior temporal gyrus.

(46) The presence of ptosis suggests damage to cranial nerve:


(A) a. IV
(B) b. V
(C) c. III
(D) d. VII

(47) A 55-year-old man complaints that he sometimes has sudden pain on one side oof
his face. It lasts up to 2 minutes. The pain is excruciating and feels like an “electric
shock”. He states it happens when he shaves and sometimes when he is chewing food.
Upon physical exam, you notice the area of the face he points to is the area of
distribution of 5th cranial nerve. He is otherwise healthy and does not take any
medications. You ask him if he has ever had shingles and says no. The most likely
diagnosis is
(A) A. Temporomandibular joint dysfunction
(B) B. Cluster headache
(C) C. Bells palsy - cranial nerve 7
(​D) D. Tic dolourex
Rationale
Tic dolourex- A chronic pain condition affecting the​ ​trigeminal nerve- cranial nerve 5 ​in
the face.
The trigeminal nerve carries sensation from the face to the brain. It's most common in
women aged over 50.
Symptoms range from mild to severe facial pain, often triggered by chewing, speaking
or brushing the teeth.
Treatment includes medication, injections and surgery.

(48) A 60-year-old man complains of extremely severe, sharp, shooting pain in his face.
Hee describes the episodes as being “like a bolt of electricity”that is brought about by
touching a specific area. Lasta about 60 seconds, and occur many times during the day.
Neurologic examination is completely normal but it is noted that part of his face is
unshaven because he fears to touch that area. Which on the following is the most
appropriate initial treatment?
(A) A. Anticonvulsants
(B) B. Aspirin
(C) C. NSAIDS
(D) D. Vasoconstrictors

(49) Which of the following function is controlled by parasympathetic system?


(A) A. Pupil constriction
(B) B. Acceleration of heart beat
(C) C. Stimulation of sweat glands
(D) D. Contraction of hair muscles

(50) . Which one of the following is NOT a probable sign of the second stage of labour?
(A) A. uterine contractions
(B) B. No cervix felt on vaginal examination
(C) C. Rupture of membranes
(D) D. Bulging of the perineum
(51) You are caring for a woman in the second stage of labor. The client is experiencing
uterine contractions every 2 minutes and cries out in pain with each contraction. This
behaviour is best described as:
(A) A. Exhaustion
(B) B. Fear of losing control
(C) C. Involuntary grunting
(D) D. Valsalva's maneuver

(52) Ms. D is admitted to the labour ward in early active labour. The priority intervention
on admission of this woman would be:
(A) A. Auscultating the fetal heart
(B) B. Taking an obstetric history
(C) C. Asking the client when she last ate
(D) D. Ascertaining whether the membranes were ruptured

(53) 4. A woman who is gravida 1, para 0 is admitted in active labour. Her cervix is 100%
effaced, and she is dilated to 3 cm. The presenting part is at +1 station. The midwife is
aware that the fetus' head is:
(A) A. Not yet engaged
(B) B. Entering the pelvic inlet
(C) C. Below the ischial spines
(D) D. Visible at the vaginal opening

(54) In order to ascertain the frequency of the woman's contractions, the midwife times
from the beginning of one contraction:
(​A) A. Until the time it is completely over
(B) B. To the end of a second contraction
(C) C. To the beginning of the next contraction
(D) D. Until the time that the uterus becomes very firm

(55) 6. A woman in labour receives a pudendal block. The midwife plans to tell the her
that once the block is working she:
(​A) A. Will not feel the episiotomy
(B) B. May lose bladder sensation
(C) C. May lose the ability to push
(D) D. Will no longer feel contractions

(56) Which one of the following foetal positions is most favourable for birth?
(A) A. Vertex presentation
(B) B. Transverse lie
(C) C. Frank breech presentation
(D) D. Posterior position of the fetal head

(57) Which one of the following assessment data can be determined by examining the
fetal heart rate strip produced by the external electronic fetal monitor?
(A) A. Gender of the fetus
(​B) B. Fetal position
(C) C. Foetal heart rate in relation to contractions
(D) D. Oxygenation

(58) Mrs. Peters is experiencing contractions every 2 minutes. She is in which stage of
birth?
(A) A. First
(B) B. Second
(C) C. Third
(D) D. Fourth

(59) 10. In which one of the following stages is the placenta delivered?
(A) A. Stage 1, latent phase
(B) B. Stage 1, active phase
(C) C. Stage2
(D) D. Stage 3

(60) Endogenous oxytocin is released from the:


(A) A. anterior pituitary gland
(B) B. cervix
(C) C. posterior pituitary gland
(D) D. uterus
(61) 12. The midwife is caring for a woman in labour. She determines that the woman is
beginning the 2nd stage of labour when which of the following signs is noted?
(A) A. The woman begins to expel clear vaginal fluid
(B) B. The contractions are regular
(C) C. The membranes have ruptured
(D) D. The cervix is dilated completely

(62) 13. The midwife is caring for a client in labor and prepares to auscultate the foetal
heart rate by using a Doppler ultrasound device. The midwife most accurately
determines that the foetal heart sounds are heard by
(A) A. Noting if the heart rate is greater than 140 BPM
(B) B. Placing the diaphragm of the Doppler on the mother abdomen
(​C) C. Performing Leopold's maneuvers first to determine the location of the fetal heart
(D) D. Palpating the maternal radial pulse while listening to the fetal heart rate

(63) Which of the following is true about congenital abnormalities?


(A) A. It refers to defects in the body structure at birth
(B) B. It is always occult at birth making it difficult to detect
(C) C. Congenital abnormalities are all inherited from the family
(D) D. It results from unknown fetal developments that are always difficult to predict

(64) When resuscitating a neonate who has neonatal asphyxia after how long should
the midwife use ambubag
(A) A. After 30 seconds of oxygen by face mask
(B) B. When the baby remains apnoeic after 30 minutes of facemask oxygen therapy.
(C) C. When the neonate is no longer cyanosed
(D) D. When the neonate P02 level has increased

(65) When should a midwife put an asphyxiated neonate on intubation and ventilation?
(A) A. When the heart beat is more than 100 beats per minute
(B) B. When the heart beat is less than 100 beats per minute
(C) C. When the heart rate has improved
(D) D. When resuscitation measures are performed without medical assistance

(66) Which neonate from the listed below is at risk of developing severe jaundice?
(A) A. Breast feed neonates
(B) B. Conjoined twins
(C) C. Neonates with haemorrhagic conditionsions
(D) D. Term neonates

(67) Which of the following is the most appropriate feature seen in symmetric growth
retarded infant?
(A) A. Weight is depressed more than length or head circumference.
(B) B. Fetal growth retardation probably has occurred in the last trimester of pregnancy.
(C) C. Weight, length and head circumference are below normal and babies look like
miniature normal babies.
(D) D. It is mostly linked with cigarette smoking.
(68) 21. A new born infant was delivered in your maternity unit an hour ago. The skin of
this infant appears to be red, wrinkly with ribs visible under the skin. When was the
infant probably born?
(A) A. Before 24 and 27 weeks
(B) B. Before 30 and 32 weeks
(C) C. Before 29 and 32 weeks
(D) D. Around 35 weeks

(69) Which statement is true about meconium?


(A) A. Meconium is 1st found in the foetal ileum between the 20th and 26th weeks of
gestation
(B) B. Meconium passage is uncommon before 36 weeks but its common in
pregnancies beyond 42 weeks
(C) C. In utero passage of meconium is common because the anal sphincter tone is
poor
(D) D. All of the above

(70) Which of the following is NOT diagnostic of twin to twin transfusion?


(A) A. Both are of same sex
(B) B. It results from venous communication of both foetuses
(C) C. Both are of different growth
(D) D. Both are of different amniotic fluid.
(71) 24. Which of the following is true about twin to twin transfusion syndrome?
(A) A. The smaller twin has the greatest risk of cardiac compromise
(B) B. The larger twin is often stuck to the uterine wall
(C) C. Serial amnia reduction improves perinatal survival
(D) D. Fetoscopic laser ablation of the placental vessels increases survival for both

(72) What would be the result of fail splitting a single fertilized ovum between 8 to 12
days after fertilization?
(A) A. Conjoined twin
(B) B. Monochorionic monoamniotic twin
(C) C. Dichorionic diamniotic twin
(D) D. Monochorionic diamniotic twin

(73) A twin pregnancy is identified at 12 weeks gestation on antenatal ultrasound. One


placenta and one amnion sac are identified. What would be the probable cause?
(A) A. ICSI with single embryo transfer
(B) B. Ovulation induction
(C) C. Maternal age over 35 years
(D) D. Family history of multiple pregnancies

(74) Which of the following best describes large for gestational age?
(A) A. Large for gestational age is weight, length, or circumference that lies above the
90th percentile for that gestational age.
(B) B. Large for gestational age is defined as birth weight more than 2 standard
deviations above the mean.
(C) C. Large for gestational age is an indication of high prenatal growth.
(D) D. Large for gestational age is weight of 3500g at birth despite gestational age.

(75) Which ONE of the following is a clinical feature clinical chorio-amnionitis?


(A) A. Foul smelling vaginal discharge
(B) B. Non tender uterus on palpation
(C) C. Maternal temperature of 37.4 degrees Celsius
(D) D. Maternal pulse of 110 beats per minute

(76) Which ONE of the following CANNOT predispose the foetus to chorio-amnionitis?
(A) A. Malnutrition
(B) B. Infection in maternal blood
(C) C. Amniocentesis
(​D) D. Abdominal ultrasound

(77) 30. Which ONE of the following is NOT a complication of preterm prelabour rupture
of membranes?
(A) A. Preterm birth
(B) B. Oligohydramnios
(C​) C. Polyhydramnios
(D) D. Infectious morbidity

(78) Which of the following BEST describes a third degree perinea! tear?
(A) A. Injury to perineum involving perineal muscles
(B) B. Injury to perineal! skin, includes the fourchette, the hymen, labia and
vaginalmucosa
(C) C. Injury to perineum involving anal sphincter complex and anal epithelium
(D) D. Injury to perineum involving the anal sphincter complex

(79) What is the greatest risk of retained products of conception in the immediate post
partum period?
(A) A. Uterine atony
(B) B. Uterine inversion
(C) C. Uterine rupture
(D) D. Uterine tenderness

(80) Which of the following is NOT an outcome for an occipito posterior position?
(A) A. Brow delivery
(B) B. Deep transverse arrest
(C) C. Persistent occipito posterior position
(D) D. Occipito anterior delivery

(81) When assessing the progress of labour, the midwife should assess for the 4 Ps.
Which ONE of the following represents the 4 Ps?
(A) A. Power, parity, passenger and passage
(B) B. Passenger, power, patient and passage
(C) C. Parity, progress, power and passenger
(D) D. Passenger, power, progress and passage

(82) What is 3rd degree moulding?


(A) A. Change of the shape of the foetal skull that occurs during labour
(B) B. Sutures of the foetal skull are overridden, but are easily separable
(C) C. Sutures of the foetal skull are in close contact
(D) D. Sutures of the foetal skull are overridden and inseparable

(83) Before performing an episiotomy, the nurse midwife needs to infiltrate the
perineum. In the ward, lignocaine 1% is available. How should the midwife dilute the
lignocaine before using it?
(A) A. Dilute 2 parts lignocaine 1% and 1 part sterile water/ normal saline
(​B) B. Dilute 1 part of lignocaine 1% and 1 part of sterile water/ normal saline
(C) C. Dilute 1 part of lignocaine 1% and 3 parts of sterile water/ normal saline
(D) D. Dilute 3 parts of lignocaine 1% and 1 part of sterile water/ normal saline

(84) Which ONE of the following is NOT an indication for an episiotomy?


(​A) A. Shoulder presentation
(B) B. Foetal distress
(C) C. Tight perineum
(D) D. Assisted vaginal delivery
(85) Which practice in labour would increase the chance of Mother to Child
Transmission of HIV in HIV positive women?
(A) A. Performing an ultrasound to rule out placental localisation
(B) B. Monitoring of maternal condition
(C) C. Foetal heart monitoring using a cardiotocograph
(​D) D. Rupture of membranes 4 hours prior to child birth

(86) In a brow presentation, what is the engaging diameter of the foetus into the
maternal pelvis?
(A) A. Sub-mento vertical diameter
(B) B. Occipito-frontal diameter
(C) C. Mento-vertical diameter
(D) D. Sub-mento bregmatic diameter

(87) Which statement BEST describes a complete breech presentation?


(A) A. Both hips are flexed and both knees are extended
(B) B. Both hips and both knees are flexed
(C) C. One or both hips are extended with the knees extended
(D) D. One or both hips are extended with the knees or knee flexed

(88) 41. Which of the following are the engaging diameters in a face presentation?
(A) A. Sub-occipitofrontal and biparietal
(B) B. Occipito-frontal and biparietal
(C) C. Mentovertical and biparietal
(D) D. Sub-mentobregmatic and biparietal

(89) Which ONE of the following IS NOT a precaution for tocolytics?


(A) A. Measure blood pressure hourly
(B) B. Give one tocolytic agent at a time
(C) C. Do not give tocolytics to women with heart rate greater than or equal to 120 beats
per minute
(D) D. Auscultate the mother's lungs every 2 hours

(90) 43. Thembi is primigravida at 39 weeks gestational age. She requested induction of
labour because she is a student in one of the universities, in preparation for her
examination. On examination:Presentation- cephalic, Fundal height- 40cm, foetal heart
rate 148 beats per minute. Before induction, the obstetrician examines her and the
findings are as follows: cervix-mid position; firm; station-3; length-2cm; and cervical
dilatation: 1-2 cm. What is Thembi's Bishop Score?
(A) 2
(B) 3
(C) 4
(D) 5
Your Answer :
B
Correct Answer :
A

(91) Which statement is FALSE concerning augmentation of labour?


(​A) A. The uterus always responds effectively to oxytocin used as administration
(B) B. Augmentation is required if labour contractions are weak and ineffective in the
absence of cephalo pelvic disproportion
(C) C. Augmentation must be gradually increased and when strong uterine contractions
occurring 3 in 10 minutes have been achieved, augmentation should be stopped
(D) D. Augmentation requires close monitoring because of the risk of uterine
hyperstimulation 45. What is cervical dystocia?

(92) What is cervical dystocia?


(A) A. When the cervix dilates at the normal expected rate during labour
(B) B. When the cervix fails to dilate in the absence of malpresentation and
cephalopelvic disproportion
(C) C. When the cervix dilates more rapidly than the expected rate
(​D) D. Failure of the cervix to dilate despite adequate uterine contractions

(93) Which of the following is NOT a contra-indication for a trial of scar?


(A) A. Multiple pregnancy
(B) B. History of sepsis following the previous caesarean section
(C) C. Previous caesarean section for cord prolapse
(D) D. Breech presentation

(94) Which ONE of the following statements best defines perinatal mortality?
(A) A. Death of a newborn in the first week of life
(B) B. Death of a newborn within the first month of life
(C) C. Stillbirth and death of a neonate in the first month of life
(​D) D. Stillbirth and death of a neonate in the first week of life

(95) Which ONE of the following drugs should be used first to a non hypertensive
woman presenting with post partum haemorrhage occurring due to uterine atony?
(A) A. Syntometrine
(B) B. Oxytocin
(C) C. Misoprostol
(D) D. Ergometrine

(96) 49. A woman presents to a clinic 7 days after normal childbirth with a fever of 37.9
Degrees Celsius and an offensive vaginal discharge. According to the Emergency
Obstetrics and Neonatal Care signal functions, where should the woman be managed?
(A) A. Clinic
(B) B. Health Centre
(C) C. Hospital
(D) D. Intensive Care Unit (ICU)

(97) Which ONE of the following statements best defines a Direct Obstetric Case
Fatality Rate?
(A​) A. The percent of women admitted with major direct obstetric complications, or who
develop such complications after admission, and die before discharge
(B) B. The percent of women admitted with direct obstetric complications, or who
develop such complications after admission, and die after discharge
(C) C. The percent of women admitted with major direct obstetric complications who
die before discharge
(D) D. The percent of women admitted with direct obstetric complications who die after
discharge
(98) Subtle neonatal seizure is recognized by:-
(A) A. Sustained eye opening or fixation, drooling, sustained posturing of a limb.
(B) B. Migrating jerks of limbs, localized clonic jerking, tonic horizontal deviation of
eyes.
(C) C. Migrating jerks of limbs, tonic horizontal deviation of eyes, lip smacking.
(D) D. Tonic horizontal deviation of eyes, apnoeic spells, lips smacking.

(99) Causes of primary amenorrhea include:-


(A) A. Gonadal dysgenesis, premature ovarian failure, asherman’s syndrome.
(B) B. Sheehan’s syndrome, gonadal dysgenesis, imperforate hymen.
(C) C. Androgen insensitivity, gonadal dysgenesis, imperforate hymen.
(D) D. Polycystic ovarian syndrome, sheehan’s syndrome, gonadal dysgenesis.
​ Rationale
● Androgen insensitivity​ syndrome (​AIS​) is when a person who is genetically male
(who has one X and one Y chromosome) is resistant to male hormones (called
androgens​). As a result, the person has some or all of the physical traits of a
woman, but the genetic makeup of a man
● Sheehan's syndrome​ is a condition that affects women who lose a
life-threatening amount of blood in childbirth or who have severe low blood
pressure during or after childbirth, which can deprive the body of oxygen. This
lack of oxygen that causes damage to the pituitary gland is known as S ​ heehan's
syndrome
● Asherman's syndrome (AS)​, is an acquired uterine condition that occurs when
scar tissue (adhesions) form inside the uterus and/or the cervix. It is
characterized by variable scarring inside the uterine cavity, where in many cases
the front and back walls of the uterus stick to one another.

(100) The following are anterior pituitary gland hormones except


(A) Prolactin
(B) Antidiuretic hormone
(C) Follicle stimulating hormone
(D) Luteinizing hormone
Rationale
Anterior pituitary
● Adrenocorticotropic hormone​ (​ACTH​)
● Thyroid-stimulating hormone​ (​TSH​)
● Luteinising hormone (​LH​)
● Follicle-stimulating hormone​ (​FSH​)
● Prolactin​ (​PRL​)
● Growth hormone​ (​GH​)
● Melanocyte-stimulating hormone (MSH)

Posterior Pituitary gland


● Oxytocin
● ADH or vasopressin

Type 4

1. Wernicke area is in the;


(A) A. Occipital lobe
(B) B. Frontal lobe
(C) C. Parietal lobe
(D) D. Temporal lobe
Rationale
Broca - inferior of frontal gyrus
Wernicke- posterior superior of temporal gyrus

(2) 2. The sensitivity part of culture and sensitivity test is for the purpose of,
(A) A. Identifying the causative organism for the infection diagnosed
(B) B. Testing the concentration of antibiotic required to cure an infection
(C) C. Identifying the most effective antibiotics against the causative agent
(D) D. Identifying the antibiotics that the patient is resistant to.

(3) 3. After a clients’ nasogastric tube has been removed the nurse should,
(A) A. Provide the client with oral hygiene
(B) B. Encourage the client to cough and deep breath
(C) C. Auscultate the client’s bowel sounds
(D) D. Offer the client liquids to drink.

(4) 4. Cardiac output is equal to;


(A) A. Heart rate × patients body weight
(B) B. Venous return× Peripheral resistance
(C) C. Peripheral resistance × stroke volume
(D) D. Stroke volume ×heart rate
Rationale

(5) 5. Bell’s palsy involves the following cranial nerve


(A) A. VIII
(B) B. VII
(C) C. V
(D) D. II
Rationale
Trigeminal nerve, cranial nerve 5 - Tic douloureux
Cranial nerve 7, Facial nerve- Bell’s palsy

(6) 6. Management of scaled lesion in psoriasis include;


(A) A. Shaving the scales using a clean razor
(B) B. Applying moisturizers to soften the scales
(C) C. Covering the lesions with occlusive dressings to remove scales
(D) D. Keeping the lesions dry to enhance dropping off the scales

(7) 7. The main causes of Diabetic keto-acidosis are;


(A) A. Infection, physical trauma, self-medication
(​B) B. Decreased dose of insulin, an illness, missed dose of insulin
(C) C. Sepsis, renal disease, liver failure
(D) D. Increased dose of insulin, starvation, use of expired insulin
Rationale
Diabetic Ketoacidosis
Definition: a complication of diabetes mellitus that is life-threatening, if not treated. It is due to
severe insulin deficiency leading to the breakdown of fats which turn into ketones. Insulin helps
to push glucose circulating in the bloodstream to the cell and severe deficiency of insulin will
cause the body to try compensate ie
1. Breaking down of stored glycogen by glucagon in the liver
2. Breaking down fats leading to ketones
Key Players of DKA:
Glucose:​ fuels the cells so it can function. However, with DKA there is no insulin present to
take the glucose into the cell…so the glucose is not used and the patient will experience
hyperglycemia ​>300 mg/dL or >17.1 mmol/L
Insulin​: helps take glucose into the cell so the body can use it for fuel. In DKA, the body isn’t
receiving enough insulin…so the GLUCOSE can NOT enter into the cell. The glucose floats
around in the blood and the body starts to think it is starving because it cannot get to the
glucose. Therefore, it looks elsewhere for energy.(breaks down glycogen in the liver, then fats)
Liver & Glucagon​: the body tries an attempt to use the glucose stores in the liver (because it
doesn’t know there is a bunch of glucose floating around in the blood and thinks the body is
experiencing hypoglycemia). In turn, the liver releases glucagon to turn glycogen stores into
more GLUCOSE….so the patient becomes even more hyperglycemic.
Ketones​: a byproduct of fat break down. In DKA, the body needs FUEL to function so it starts to
break down FATS since it cannot use the glucose in the body. The patient will experience
increased ketones in the body which are LIFE-THREATENING to a diabetic patient because it
causes the blood to be very acidic (​metabolic acidosis!)
Kidneys​: plays a role in reabsorbing glucose in the renal tubules. However, there is too much
glucose present in the blood and it cannot be reabsorbed. So, it leaks into the urine and this
causes OSMOTIC DIURESIS which causes polyuria and excretion of electrolytes
(sodium,potassium, chloride)
Causes of DKA
1. Undetected diabetes​: patient doesn’t know they are diabetic and this is the first sign,
usually.
2. More Insulin needed by the body than normal:​ the body needs more units of insulin
than it is actually receiving from injections. Example: when a diabetic become sick
(INFECTION) with illness or recovering from surgery or experiences some type of stress
(stress leads to release of cortisol). Also certain drugs such as, corticosteroids or
thiazide diuretics cause increased need for insulin.
3. Not eating (skipping meals): ​body starts to go into “starvation” mode and begins to
burn ketones (normally in nondiabetics when the body goes into starvation mode it can
cope when ketones are released by regulating insulin and glucagon to maintain sugar
levels…but in the diabetic they don’t have that ability and ketones production is
dangerous).
4. Not taking insulin as scheduled​: therefore the blood glucose levels are not
controlled…ketones are produced and the cycle of acidosis starts to take place in the
body.
Signs & Symptoms of Diabetic Ketoacidosis:
What is going on in a patient with DKA:
1. Hyperglycemia ​(intracellular to extracellular shifting takes place which will lead
to electrolyte imbalances)
2. Ketones ​in the blood (leads to metabolic acidosis, weight loss because of all the
fat burning, electrolyte shifting as well)
3. Metabolic Acidosis​ (blood pH <7.35 and HCO3 <15 mEq/L)
4. Polyuria​: due to the extreme levels of glucose in the body that causes the water
inside the cells to shift to the extracellular area. The kidneys try to compensate by
increasing urinary production to eliminate this extra fluid but the kidneys cannot
reabsorb all the glucose so it leaks into the urine. This causes OSMOTIC
DIURESIS which causes SODIUM AND POTASSIUM (along with calcium, phos)
TO BE EXCRETED.
NOTE potassium levels typically stay normal or elevated in DKA because of
the shifting of potassium from the inside of the cell to the outside BUT
WHEN TREATMENT STARTS TO BE INITATED WITH INSULIN IT WILL
CAUSES THE K+ TO MOVE BACK INTO THE CELL. Therefore, you have to
watch POTASSIUM LEVELS closely during treatment.
5. Polydipsia​: frequent drinking due to extreme thirst….vicious cycle of frequent
urination and the body is trying to keep itself hydrated.
6. Dehydration:​ dry mucous membranes, decreased skin turgor (the extreme
drinking doesn’t work)
7. Nausea & vomiting, Abdominal pain-​> (especially children…causes not 100%
known but could be due to the ketones present in the blood)
8. Kussmaul Breathing: ​due to metabolic acidosis….the respiratory system tries to
compensate by getting rid of extra acid in the body by blowing off carbon dioxide
which is an acid…this is rapid deep breathing
9. Acetone Smell of the Breath “fruity​”: due to the breakdown of ketones
10. Ketones​ present in the urine
11. Tachycardia, hypotension, confusion, fatigue

Nursing Interventions of DKA


1. Teach​ patient early signs and when to seek treatment: This is a medical emergency!!-
Let them know they need to get treatment early because DKA is fatal
2. Monitor ​glucose and ketones during illness every 4 hours, especially if dealing with
illness/infection
3. Hydration​ - If vomiting and cannot eat food or drink liquids notify doctor (if can tolerate
drink liquids every hour)
4. Notify ​medical doctor if blood sugars are higher than normal or greater than 300 mg/dL
or greater than 17.1 mmol/L consistently
Management/Treating DKA
Goal: Hydrate, decrease blood glucose, monitor potassium level and cerebral edema (esp. in
children), correct acid-base imbalance
1. Administer IV fluids:​ (depending on MD order) such as 0.9% normal saline (start out
with a bolus of this) and progress with 0.45% NS ​to hydrate the cells ​(depends on how
dehydrated the patient is)
5% dextrose may be added to the 0.45% NS when glucose is around 250 to 300 mg/dL.
This will help gradually bring the blood sugar down and help the insulin do its job by
removing the ketones.
2. Administer insulin​: REGULAR (only type given IV) and make sure K+ is normal >3.3
Typically started out by giving unit IV bolus…then start an infusion (checking blood
glucoses around the clock…hospital protocols)…you will be titrating the insulin base on
blood glucose checks.
NOTE: if you rapidly bring a patient’s blood glucose down (or up) the brain can’t cope
and water will be moved from the blood to the CSF and you will get cerebral edema and
increased intracranial pressure
Tip for insulin administration: when priming tubing for insulin infusion waste 50cc to
100cc (per institution protocol) because insulin absorbs into the plastic lining of the
tubing.
3. Monitor potassium levels very closely ​because insulin causes K+ to move back into
the cell. Administer Potassium solution IV to combat this….note renal function before
administering and phlebitis
(8) 8. The following is true regarding Hashimoto thyroiditis
(A) A. Is an autoimmune disease, causes primary hypothyroidism
(B) B. Is an autoimmune disease, causes primary hyperthyroidism
(C) C. Results in goiter, causes exophthalmos
(D) D. More common in women, there is an increase in TH levels.
Rationale
➢ Hashimoto’s disease​ is an a ​ utoimmune disorder​ that can cause h​ ypothyroidism​,
or underactive thyroid. With this disease, your i​ mmune system​ attacks your
thyroid. The thyroid becomes damaged and can’t make enough thyroid
hormones​.The thyroid is a small, butterfly-shaped gland in the front of your neck.
Thyroid hormones control how your body uses energy, so they affect nearly every
organ in your body—even the way your heart beats. Without enough thyroid
hormones, many of your body’s functions slow down.
➢ Graves' disease​ is an autoimmune disorder that causes hyperthyroidism, or
overactive thyroid. With this disease, your immune system attacks the thyroid
and causes it to make more thyroid hormone than your body needs. The thyroid
is a small, butterfly-shaped gland in the front of your neck.
● Exophthalmos (​ also called exophthalmos, exophthalmia, proptosis, or
exorbitism) is a bulging of the eye anteriorly out of the orbit.
Exophthalmos can be either bilateral (as is often seen in Graves' disease)
or unilateral (as is often seen in an orbital tumor).

(9) 9. All the following nursing diagnoses are appropriate for a client with acute asthma
attack, which is of the highest priority?
(A) A. Anxiety related to difficulty in breathing
(​B) B. Ineffective airway clearance related to bronchoconstriction and increased mucus
production
(C) C. Ineffective breathing pattern related to discomfort during breathing
(D) D. Ineffective health maintenance related to lack of knowledge about attack triggers
and appropriate use of medication

(10) 10. Russell’s traction is indicated in a;


(A) A. Femur, knee and hip fractures
(B) B. Femur, acetabulum and lower leg fractures
(C) C. Minor fractures of lower spine, sciatica, muscle spasms
(D) D. Degenerative disc disease of the cervical spine, shoulder dislocation, muscle
spasms
Rationale
(11) 11. Characteristics of 2nd degree burns include;
(A) A. Waxy white dry, leathery, charred, oedema
(B) B. Fluid-filled blisters, weeping of fluid, bright pink
(C) C. Red to gray, may have local edema
(D) D. severe pain, weeping of fluid, may have local edema, leathery charred
Rationale
● 1st-degree burn.​ This minor burn affects only the outer layer of the skin
(epidermis). It may cause redness and pain.
● 2nd-degree burn.​ This type of burn affects both the epidermis and the
second layer of skin (dermis). It may cause swelling and red, white or
splotchy skin. Blisters may develop, and pain can be severe. Deep
second-degree burns can cause scarring.
● 3rd​-​degree burn​. ​This b
​ urn​ reaches to the fat layer beneath the skin.
Burned areas may be black, brown or white. The skin may look leathery.
Third-​degree burns​ can destroy nerves, causing numbness

(12) 12. When estimating the size of burns, the palm of the patient’s hand represents
body surface area of;
(A) A. 3%
(B) B. 1%
(C) C. 4.5%
(D) D. 9%
(13) The early signs of cancer of the posterior nasal space include;
(A) A. Proptosis, hearing loss
(B) B. Hearing loss, double vision
(C) C. Purulent nasal discharge, aphonia
(D) D. Anosmia, proptosis

(14) A client complains of decreasing peripheral vision and halos around lights. These
manifestations are characteristics of;
(A) A. Retinal detachment
(B) B. Open-angle glaucoma
(C) C. Cataract
(D) D. Macular degeneration
Rationale
● Strabismus​ - patch stronger eye
● Foreign object in the eye-​ Do not remove. Cover both eyes with paper cups
● Retinal detachment ​- Seeing small ​flashes of light​ is associated with retinal
detachment.
● Tunnel vision and halos​ - seen in glaucoma
● Macular degeneration​ - loss of central vision
● Macular Degeneration
Macular degeneration is a progressive, incurable disease of the eye in which the
central portion of the retina, the macula, begins to deteriorate. This deterioration
causes distortion (blurred or wavy visual disturbances) or loss of the central field
of vision, whereas the peripheral vision remains intact.Macular degeneration has
two different etiologies. "Dry" macular degeneration involves ischemia and
atrophy of the macula that results from blockage of the retinal microvasculature.
"Wet" macular degeneration involves the abnormal growth of new blood vessels
in the macula that bleed and leak fluid, eventually destroying the macula.
Progression of macular degeneration may be slowed with smoking cessation,
intake of specific supplements (eg, carotenoids, vitamins C and E), laser therapy,
and injection of antineoplastic medication​s. ​Risk factors for macular
degeneration​ include ​advanced age, family history, hypertension, smoking, and
long-term poor intake of carotenoid-containing fruits and vegetables.
​Retinal detachment​-curtain appearing vision ​ ​Macular degeneration​ -loss of central vision
Glaucoma-​ Tunnel vision and halos Astigmatism/Cataracts -blurry vision

(15) 15. Malena can be caused by;


(A) A. Small bowel obstruction, abdominal aortic aneurysm, internal hemorrhoids
(B) B. Perforated gastric ulcers, neoplasms in the transverse colon
(C) C. Bleeding esophageal varices, perforated gastric ulcer
(D) D. Abdominal aortic aneurysm, splenic neoplasm, esophageal varices

(16) 16. Which sign and symptoms should a nurse expect to find in a client diagnosed
with ulcerative colitis?
(A) A. Twenty bloody stools a day
(B) B. Chronic constipation
(C) C. Oral temperature of 39o c
(D) D. Hard rigid abdomen

(17) 17. Early miscarriage refers to;


(A) A. loss of pregnancy spontaneously in between 16-24 weeks
(B) B. spontaneous loss of pregnancy from between 16-30 weeks gestation
(C) C. spontaneous loss of pregnancy from between 0 to 16 weeks gestation
(D) D. loss of pregnancy between 24 to 30weeks

(18) 18. The patient was brought to the outpatient department with a cut wound, which
is the most appropriate intervention for a triage nurse;
(A) A. Wash the site with soap and water
(B) B. Squeeze the site to enhance bleeding
(C) C. Apply a tourniquet proximal to the site
(D) D. Apply a tourniquet distal to the site
(19) 19. Bacterial growth phase where cellular division seems to have stopped or
ceases for a period of time:
(A) A. The Lag Phase
(B) B. The Log Phase
(​C) C. The Stationary Phase
(D) D. The Death Phase

Rationale

​ ag phase: ​After a liquid culture broth is inoculated, themultiplication of bacteria


L
does not start immediately. It takes some time to multiply. The time between
inoculation and beginning of multiplication is known as lag phase. In this phase,
the inoculated bacteria become acclimatized to the environment, switch on various
enzymes, and adjust to the environmental temperature and atmospheric conditions.
During this phase, there is an increase in size of bacteria but no appreciable
increase in number of bacterial cells. The cells are active metabolically. The
duration of the lag phase varies with the bacterial species, nature of culture
medium, incubation temperature, etc. It may vary from 1 hour to several days.
2. Log phase: ​This phase is characterized by rapid exponentialcell growth (i.e., 1
to 2 to 4 to 8 and so on). The bacterial population doubles during every generation.
They multiply at their maximum rate. The bacterial cells are small and uniformly
stained. The microbes are sensitive to adverse conditions, such as antibiotics and
other antimicrobial agents.
3. ​Stationary phase: ​After log phase, the bacterial growth almoststops completely
due to lack of essential nutrients, lack of water oxygen, change in pH of the
medium, etc. and accumulation of their own toxic metabolic wastes. Death rate of
bacteria exceeds the rate of replication of bacteria. Endospores start forming
dur-ing this stage. Bacteria become Gram variable and show irregular staining.
Many bacteria start producing exotoxins.
4. Decline phase: ​During this phase, the bacterial populationdeclines due to death
of cells. The decline phase starts due to (​a)​ accumulation of toxic products and
autolytic enzymes and (​b​) exhaustion of nutrients. Involution forms are common in
this stage. Growth rate during different phases of bacterial growth curve is
summarized in Table 2-5.

20) 20. A nurse is showing a patient how to use crutches at home and instructing his
mother about how to change his bandages. At this time, the nurse is primarily acting as:
(A) A. Patient advocate
(B) B. Manager of care
(C) C. Teacher
(D) D. Decision maker

(21) 22. While caring for a patient with a diagnosis of emphysema, the healthcare
provider instructs the patient on positioning that will assist the patient’s ability to
breathe when experiencing dyspnea. Which of the following positions will the healthcare
provider instruct the patient about?
(A) A. Lie supine with your feet on a blanket roll.
(B) B. Sit upright and lean forward on a supportive surface
(C) C. Sit in a recliner chair at a 120-degree angle
(D) D. Assume a side-lying position
(22) 23. During chest auscultation of a patient, which findings will indicate that the
patient has cardiac condition and cannot be felt in a normal person
(A) A. S1, S3 , Murmurs
(B) B. S1, S2, Negative Murmurs,
(C) C. S3, S4, Murmurs
(D) D. S3, S1,S2

(23) 24. Nurse Lei caring for a client with a pneumothorax and who has had a chest tube
inserted notes continues gentle bubbling in the suction control chamber. What action is
appropriate?
(A) A. Do nothing, because this is an expected findng
(B) B. Immediately clamp the chest tube and notify the physician
(C) C. Check for an air leak because the bubbling should be intermittent
(D) D. Increase the suction pressure so that the bubbling becomes vigorous

(24) 25. You assess a client with newly diagnosed hypothyroidism as having an
enlarged thyroid gland (goiter). What pathologic process causes this enlargement?
(A) A. An excess of TH stimulates thyroid follicles
(B) B. An increased dietary iodine intake
(C) C. A compensatory effort to produce more TH
(D) D. Tissue hypertrophy in response to increased TH
Rationale
When T4 enters the circulation, it gets converted to T3 through the process of
deiodination. T4 and T3 can then exert negative feedback on TSH levels (high levels of
T3/T4 decrease TSH release from the anterior pituitary, while low levels of T3/T4
increase TSH release). T3 is the predominant inhibitor of TSH secretion. Because TSH
secretion is so sensitive to minor changes in free T4 through this negative feedback
loop, abnormal TSH levels are detected earlier than those of free T4 in hypothyroidism
and hyperthyroidism.

(25) 26. An elderly client has severe xerosis. What topic should be included in your
teaching plan?
(A) A. Maintain a warm environment
(B) B. Take a hot bath everyday
(C) C​. Apply skin lotions after a bath
(D) D. Use fabric softeners when laundering clothing
(26) 27. Which of the following clients would be most at risk for the development of
type 2 diabetes?
(A) A. Middle-age man who maintains normal weight
(B) B. Young adult who is a professional basketball player
(C) C. Middle-aged woman who is the sole caretaker of her parents
(D) D. W
​ oman over the age 70 who is overweight and sedentary

(27) 28. A client received a liver transplant a day ago. If a client were to develop an
acute transplant rejection episode, when should the nurse expect to see the
manifestations?
(A​) A. Approximately 4days to 3 months later
(B) B. Approximately 2 days later
(C) C. Within the first 24 hours
(D) D. Within the first 8 hours

(28) Ms. Smith complains of nausea and vomiting following her daily chemotherapy
treatment. The most appropriate nursing intervention would be to;
(A) A. Keep NPO until her daily chemotherapy is completed
(B) B. Provide antiemetic medication 30 to 40 minutes prior to each treatment
(C) C. Provide clear liquids until the chemotherapy is completed
(D) D. Schedule chemotherapy administration to bed time

(29) 21. The nurse in charge identifies a patient’s responses to actual or potential health
problems during which step of the nursing process?
(A​) A. Assessment
(B) B. Diagnosis
(C) C. Planning
(D) D. Evaluating

(30) 30. Which of the following clients is a greater risk for burn shock?
(A) A. One with 90% superficial burns from a tanning bed
(B) B. One with 10% TBSA from gasoline explosion
(C) C. One with radiation burns following treatment for cancer
(D) D. One with >50% TBSA from a high-voltage electric accident

(31) 31. Which one of the following topics should be included in a health message on
burn prevention?
(A) A. Use a solar powered night light
(B) B. Check smoke detectors annually
(C) C. Do not use oven for cooking
(D) D. Set water heater no longer than 48°C

(32) While providing care to an old woman with a cast on her lower arm (from below the
elbow the elbow to above the fingers) you perform a neurovascular assessment. What
is the possible complication?
(A) A. Slightly edematous fingers
(B) B. Warm, pink skin above the cast
(C) C. Pale, cold fingers
(D) D. Pain rating of 2 on a 1 to 10 scale

(33) 33. A patient becomes restless and tells you she has a headache and feels
nauseous during hemodialysis. Which complication do you suspect?
(A) A. Infection
(B) B. Disequilibrium syndrome
(C) C. Air embolism
(D) D. Acute hemolysis
Your Answer :

(34) 34. A patient Kante, 16 years old is brought to the Emergency department with
history of sodium hypochloride/JIK spill on the left eye 5 minutes ago. The first most
appropriate Nurse action would be?
(A) A. Immediately refer to an ophthalmologist for review
(B) B. Administer topical antibiotics and analgesics
(C) C. Test for Visual Acuity to check extent of damage
(D) D. Patch the eye after irrigation with clean water

(35) 35. A classification of ringworm that infect the finger nails is:-
(A) A. Tinea coporis
(B) B. Tinea unguium
(C) C. Tinea cruris
(D) D. Tinea pedis

(36) 36. Following gastrectomy, dumping syndrome occurs as a results of:-


(A) A. Reduced gastric motility resulting from Vagus nerve stimulation
(B) B. Removal of acid producing cells thus resulting in reduced absorption of Vit B12
(C) C. Rapid gastric emptying of hyperosmolar fluid and substances into the duodenum
(D) D. Damage of Vagus nerve and cardiac sphincter
(37) 37. The intracranial; hemorrhage with the worst prognosis is :-
(A) A. Cephalohematoma
(B) B. Epidural hemorrhage
(C) C. Intracerebral hemorrhage
(D) D. Subarachnoid hemorrhage

(38) 38. Signs of hypothyroidism include:-


(A​) A. Muscle weakness, depression
(B) B. Tachycardia, irritability
(C) C. Decreased appetite, tachycardia
(D) D. Hypertension, somnolence

(39) 39. A patient with potassium levels in excess of 5.0 mEq/L is considered to be in a
state of :-
(A) A. Hypernatremia
(B) B. Hypercalcemia
(C) C. Hypervolemia
(D) D​. Hyperkalemia

(40) 40. The health message given to a patient following tympanoplasty is:-
(A) A. Drink fluids using a straw
(B) B. Avoid blowing the nose and sneeze through the mouth
(C) C. Administered saline water into the ear to aid healing
(D) D. Avoid noisy places upto two weeks post operatively

(41) 41. Which of the following indicates a normal finding on percussion of the lungs
(A) A. Tympani over the right upper lobe
(B) B. Resonance over the left upper lobe
(C) C​. Hyper resonance over the left lower lobe
(D) D. Dullness above the left 10th intercostal space

(42) 42. What are the causes of ischaemic stroke


(A) A. Saccualar aneurysm, artery thrombosis, large artery thrombosis
(B) B. Cardiogenic embolism, small artery thrombosis, cryptogenic
(C) C. small artery thrombosis, cryptogenic, intracerebral hemmorage
(D) D. Arterio-venous malformation, Cardiogenic embolism, small artery thrombosis

(43) 43. Which one of the following is correct regarding the control of glucose levels?
(A) A. Both insulin and glucagon are necessary to maintain high blood glucose levels.
(B) B. Glucagon is responsible for switching off gluconeogenesis.
(C) C. Insulin is responsible for switching on glycogenolysis.
(D) D. Glucagon is responsible for switching on gluconeogenesis

(44) 44. What type of stool can a nurse expect from a client who has a colostomy of the
lower descending colon?
(A) A. Liquid
(B) B. Bloody
(C) C. Black
(D) D.​ Formed

(45) 45. An adult patient weighing 120 kg has a 20% total body surface area burn. Using
the modified parkland formula, how much total intravenous fluid should he receive in
the first 8 hours:-
(A) A. 2400 mL
(B) B. 4800mL
(C) C. 9600 mL
(D) D. 10,800 ml
4ml x 20 x 120kg = 9600 ml Total to be given in 24hrs
First half, 4800 ml given in the first 8hrs
Second half given in next 16hrs

(46) 46. Antihistamines are indicated in the management of vertigo because they:
(A) A. Sedate the client
(B) B. Activate the vestibular system
(C) C. Suppress the vestibular system
(D) D. Suppress the cochlear system

(47) 47. Clinical features of hydatidiform mole include;


(A) A. Positive pregnancy test of urine diluted to 1:100 after4 weeks of amenorrhea,
ultrasound shows features of fetus.
(B) B. Amenorrhea followed by vomiting, breast changes.
(C) C. Uterus larger than dates ,chronic gonadotropin hormone in urine increase
(D) D. Amenorrhea followed by intermittent bleeding, fetus demonstrated in x-ray.
(48) 48. Glomerulonephritis usually follows;
(A) A. Staphylococcal infection
(B) B. Gonococcal infection
(C) C. Hemophilus infection
(​D) D. Streptococcal infection

(49) 49. The aorta divides at the level of the 4th lumbar vertebra into;
(A) A. external &internal iliac arteries
(B) B. superior & inferior mesenteric arteries
(C) C. common iliac arteries
(D) D. lumbar arteries

(50) 50. Corneal infections are difficult to treat due to the fact that;
(A) A. The surface area of cornea is small
(B) B. Causative organisms cannot be recovered
(C) C. Corneal infections are not easily diagnosed
(D)​ D. The cornea is avascular organ

(51) 51. The most appropriate intervention for a client with a diagnosis of ineffective
airway clearance is:
(A) A. Withholding pain medications
(B) B. Administering cough suppressants every 4-6 hours
(C) C. Administering antibiotics as prescribed
(D) D. Maintaining adequate hydration and providing increased humidity.

(52) 52. Chvostek’s sign is used for diagnosis of:-


(A) A. Hypercalcemia resulting from hyperpituitariism
(B) B. Hypocalcemia resulting from hypoparathyroidism
(C​) C. Hypocalcemia resulting from hypothyroidism
(D) D. Hypocalcemia resulting from hypothyroidism

(53) 53. Which is a symptom of atopic dermatitis?


(A) A. A blister on the palms of hands and soles of feet.
(B) B. Itchy, inflamed skin.
(C) C. Scaly patches of skin on the scalp.
(D) D. Coin-shaped patches of irritated skin on the arms and lower legs.

(54) 54. A primary defect in osteomalacia is a deficiency in which vitamin?


(A) A. B12
(B) B. D
(C) C. E
(D) D. C

(55) 55. Obesity is a risk factor for all of the following types of cancer except:-
(A) A. Renal
(B) B. Pancreas.
(C) C. Endometrial.
(​D) D. Lung cancer.

(56) 56. A child on long term use of corticosteroids is at risk of;


(A) A. Growth retardation.
(B) B. Peripheral neuropathy.
(C) C. Muscular degeneration.
(D) D. Hyperkalemia.

(57) 57. Pleural effusion is a complication of;


(A) A. Heart failure, nephritic syndrome, pulmonary tuberculosis
(B) B. Pneumonia,upper respiratory infections, post thoracotomy
(C) C. Penetrating chest trauma,bronchitis,haematogenous infection of the pleural
space
(D) D. Pulmonary embolism, lung cancer, post thoracentesis

(58) When measuring central venous pressure (CVP) the nurse should;
(A) A. Mark the location of the right atrium.
(B) B. Identify the location of the left atrium.
(C) C. Always ensure the patient lies in supine position.
(D) D. Allow Normal saline to drip rapidly into the client for 5 min before taking CVP
reading

(59) 60. Furuncles (boils) are commonly caused by;


(A) A. Streptococcus pyogenes
(B) B. Clostridium tetani
(C) C. Staphylococcus aureus
(D) D. Bacillus aureus

(60) 61. The appropriate health message to a patient who develops stomatitis
secondary to radiotherapy is ;
(A) A. G
​ urgle with mouthwash and rinse thoroughly after each meal.
(B) B. Use ice cold liquids such as tea or cola to relieve discomfort.
(C) C. Use a toothbrush soaked in saline to clean the mouth.
(D) D. Drink citrus juices

(61) 62. Sympathetic stimulation of the cardiovascular system causes;-


(A) A. Increased heart rate and constriction of coronary arteries.
(B) B. Constriction of GIT secretory gland vessels with increased flow of digestive
juices.
(​C) C. Increased peripheral resistance and dilatation of coronary arteries​.
(D) D. Increased force of heart contraction & reduced peripheral resistance.

(62) 63. The immediate effects of burns on body fluids and electrolytes include ;-
(A) A. Hypokalemia & Hyponatremia.
(B) B. Reduced circulatory blood volume and hypernatremia.
(C) C
​ . Hyponatremia and Hyperkalemia.
(D) D. Reduced urine output and hypokalemia

(63) 64. Early complication of fractures include;


(A) A. Shock, vascular necrosis.
(B) B. Reaction to internal fixation device, deep venous thrombosis
(C)​ C. Compartment syndrome, fat embolism.
(D) D. Complex regional pain syndrome, infection.

(64) 65. When suctioning a laryngeal tube;-


(A) A. It should take about 10 seconds.
(B) B. A clean and not asceptic technique should be used.
(C) C. Suction is applied while inserting the catheter into the tube.
(D) D. Suctioning should be done continuously till secretions clear.
Your Answer :
D
Correct Answer :
B
(65) 67. Angiotensin-converting enzyme (ACE)inhibitors;
(A) A. Inhibit conversion of angitensinogen to angiotensin I.
(B) B. Block receptors from binding angiotensin II.
(C) C. Inhibit conversion of angiotensin I to angiotensin II.
(D) D. Block angiotensin I receptor sites.
Your Answer :
C
Correct Answer :
C
(66) 68. In myocardial infarction, Nitroglycerin is administered to;-
(A) A. Dilate arteries in small doses.
(B) B. Decrease ischemia by reducing myocardial oxygen consumption.
(C) C. Increase the preload.
(D) D. Enable the patient achieve muscle relaxation.
Your Answer :
A
Correct Answer :
A
(67) 69. After a barium enema a nurse should,
(A) A. Keep the patient NPO for 8 hours to rest the bowel
(B) B. Advise the patient that his/her stool will be radioactive
(C) C. Encourage the patient to increase his fluid intake
(D) D. Advise the patient to avoid abdominal exercises for at least 1 week
Your Answer :
C
Correct Answer :
C
(68) 70. Passive range of motion exercises;
(A) A. Are performed to prevent nosocomial infections.
(B) B. Helps to prevent joints rigidity and contractures.
(C) C. Is a collaborative intervention that requires a surgeon and a nurse.
(D) D. Are repeated 5-6 times a day to enhance quick recovery.
Your Answer :
D
Correct Answer :
B
(69) 71. Appropriate nursing interventions for a patient with “Risk for infection related to
altered immunity response” include;
(A) A. Barrier nursing, avoiding invasive procedures and inspection of entry port sites for
pathogens
(B) B. Reverse barrier nursing, monitor SPO2 hourly and serve high protein diet
(C) C. Isolation nursing, obtaining cultures & sensitivities daily and administration of
antibiotics
(D) D. Reverse barrier nursing, avoiding invasive procedures & daily assessment of
intravenous sites
Your Answer :
D
Correct Answer :
D
(70) 72. The Korotkoff sounds heard while measuring blood pressure present in the
order of,
(A) A. Tapping, Swishing, Knocking, muffling and silence
(B) B. Swishing, Tapping, Silence, muffling and knocking
(C) C. Knocking, Tapping, Swishing, muffling, silence
(D) D. Silence, Tapping, Knocking, Swishing and Muffling
Your Answer :
B
Correct Answer :
A
(71) 73. In sickle cell anaemia patients,
(A) A. The structure of haemoglobin molecule is not affected.
(B) B. Prophylactic administration of folic acid and pneumococcal vaccine is
recommended
(C) C. Administration of IV fluids & immediate blood transfusion are encouraged during
sickle cell crisis situations
(D) D. The treatment of choice includes administration of penicillins & NSAIDs
Your Answer :
C
Correct Answer :
C
(72) 74. Hyperkalemia commonly occurs in the following,
(A) A. Burns, pneumonia and congestive cardiac failure.
(B) B. Renal failure, crush injuries and burns.
(C) C. Digitalis toxicity, DKA and hyperthyroidism.
(D) D. Overuse of potassium sparing diuretics, CCF and liver cirrhosis
Your Answer :
C
Correct Answer :
B
(73) 75. The route that achieves the quickest absorption and effects of a drug is the,
(A) A. Subcutaneous
(B) B. Intradermal
(C) C. Intramuscular
(D) D. Oral
Your Answer :
D
Correct Answer :
C
(74) 76. In simple mastectomy:-
(A) A. Pectoralis major muscle and the entire breast is excised
(B) B. Axillary lymph nodes and pectoralis major muscles are left intact
(C) C. Underlying chest muscles and chaniin of internal mammary lymph nodes are
removed
(D) D. Entire breast and axillary lymph nodes are removed
Your Answer :
B
Correct Answer :
B
(75) 77. Reed Sternberg cell is a pathologic hallmark essential in diagnosis of:-
(A) A
​ . Hodgkin’s disease
(B) B. Acute myeloid leukemia
(C) C. Chronic lymphocytic leukemia
(D) D. Non-hodgkin disease

(76) 78. Characteristics of 1st degree burns include:-


(A)​ A. Erythema and pain
(B) B. Chilling and dry wounds
(C) C. Healing by scaling, painless wounds
(D) D. Black appearance, painless wounds

(77) 79. In rheumatoid arthritis, inflammation of the jounts begins in the:-


(A) A. Hip, ankle,elbow
(B) B. Feet, wrist, toes
(C) C. Shoulder, fingers, wrist
(D) D. Clavicle, hands, fingers
Your Answer :
D
Correct Answer :
C
(78) 80. The main aim of escharotomy in treatment of full thickness burns is to:-
(A) A. Improve oxygenation of the burnt tissues
(B) B. Remove the dead tissues
(C) C. Allow underlying viable tissues to expand
(D) D. Reduce pain
Your Answer :
C
Correct Answer :
C
(79) 81. Risk factors of osteoporosis include:-
(A) A. Lack of Vitamin D in diet, high estrogen levels in women
(B) B. Too much exposure to the sun, low testosterone levels in men
(C) C. Increase in age, late menopause
(D) D. Excessive alcohol consumption, early menopause
Your Answer :
Correct Answer :
D
(80) 82. Rhesus factor is found in the cell membrane of the:-
(A) A. Lymphocytes
(B) B. Granulocytes
(C) C. Erythrocytes
(D) D. Globulins
Your Answer :
C
Correct Answer :
C
(81) 83. The immediate action in a patient with sickle cell anaemia who develops
priapism is:-
(A) A. Caution the patient against passing urine till the priapism resolves
(B) B. Make the patient to lie still on bed
(C) C. Have the patient take a warm bath
(​D) D. Administer morphine at the onset of the attack
Your Answer :
A
Correct Answer :
D
(82) 84. Atherosclerosis impedes coronary blood flow by the following mechanisms
(A) A. Plaques obstructs the vein
(B) B. Plaques obstruct the artery
(C) C. Blood clots from outside the vessel wall
(D) D. Hardened vessels dilate to allow the blood to flow through
Your Answer :
B
Correct Answer :
B
(83) 85. The hypogastric region of the abdominopelvic cavity is;
(A) a. Inferior to the umbilical region, medial to the right iliac region
(B) b. Lateral to the left iliac region, superior to the umbilical region
(C) c. Medial to the right iliac region, lateral to the umbilical region
(D) d. Superior to the umbilical region, medial to the right iliac region
Your Answer :
D
Correct Answer :
A
(84) 86. The most appropriate nursing action when a client begins to cough and has
difficulty breathing during nasogastric tube insertion is;
(A) A. Inserting the tube quickly
(B) B. Notifying the physician immediately
(C) C. Removing the tube and reinserting when the respiratory distress subsides
(D) Pulling the tube back a little and wait until the respiratory distress subsides
Your Answer :
D
Correct Answer :
D
(85) 87. Turbid urine is an indication of;
(A) a. Dehydration and urinary tract infection.
(B) b. Presence of prostatic fluid and WBC.
(C) c. Anuria and presence of RBCs.
(D) d. Perineal injury and prolonged use of diuretics
Your Answer :
A
Correct Answer :
A
(86) 88. An 11-month-old infant with dehydration and metabolic acidosis is likely to
present with;
(A) a. A decreased platelet count
(B) b. Shallow respirations
(C) c. Tachypnea
(D) d. A reduced white blood cell count
Your Answer :
B
Correct Answer :
C
(87) 89. A child with Wilms’ tumor presents with;
(A) a. Gross hematuria
(B) b. Dysuria
(C) c. Nausea and vomiting
(D) d. An abdominal mass
Your Answer :
D
Correct Answer :
D
(88) 90. The expected outcomes for a patient who has “potential for impaired gas
exchange related to fluid overload” is;
(A) a. No shortness of breath, scaphoid abdomen.
(B) b. Chest X-ray normal, improved gas exchange.
(C) c. Respiratory rate less than 20breaths/min, skin colour normal.
(D) d. Heart rate less than 100 beats/min, increased urine output
Your Answer :
B
Correct Answer :
D
(89) 91. During Lumbar puncture, a needle is inserted in the space between;-
(A) a. Fifth and sixth thoracic vertebrae.
(B) b. Fourth and fifth thoracic vertebrae
(C) c. Third and fourth lumbar vertebrae.
(D) d. First and second lumbar vertebrae
Your Answer :
C
Correct Answer :
C
(90) 92. A deteriorating condition for a patient with head injury will be indicated by;-
(A) a. Widening pulse pressure and irregular respiratory pattern.
(B) b. Narrowing pulse pressure and escalation of discomfort.
(C) c. Bradycardia and Kussmaul breathing.
(D) d. Oliguria and narrow pulse pressure
Your Answer :
C
Correct Answer :
A
(91) 93. In chronic bronchitis;-
(A) a. There is cough and sputum production for at least 3 months in 2 consecutive
years.
(B) b. There is destruction of alveoli walls due to severe infection and increased
secretion.
(C) c. There is increased ciliary function in the airway.
(D) d. There is flow limitation and is not fully reversible

(92) 94. The condition in which the eyeballs are not properly aligned with each other is
referred to as;
(A) a. Astigmatism
(B) b. Strabismus
(C) c. Presbyopia
(D) d. Amblyopia

(93) 95. A patient with diabetic ketoacidosis presents with hyperventilation because;
(A) A. The respiratory centre in the brain is damaged
(B) B​. The body PH is low
(C) C. There is high rate of oxygen consumption
(D) D. There is reduced excretion of waste through the renal system

(94) 96. When assessing a client with possible Cushing’s syndrome you would expect to
find;
(A) A. Hypotension
(B) B. Thick coarse skin
(C) C
​ . Deposits of adipose tissue in the trunk
(D) D. Weight loss

(95) 97. When administering an enema;


(A) A. The client should lie in the right lateral position.
(B) B. The enema tube should be inserted 8 inches into the clients’ rectum.
(C) C. The client should be advised to retain the enema for 30 minutes.
(D) D. The nurse should position the client in left lateral position

(96) 98. The primary causes of acute otitis media are;-


(A) A. Escherichia coli and proteus spp.
(B) B. Streptococcus pneumoniae and Haemophilus influenzae.
(C) C. Staphylococcus aureus and haemophilus influenzae.
(D) D. Proteus spp. and Streptococcus pneumoniae.
(97) 99. Triple therapy is indicated in treatment of;
(A) A. Osteomyelitis.
(B) B.​ Helicobacter pylori.
(C) C. Neisseria meningitidis.
(D) D. Oncocerca volvulus

(98) 100. The early signs of cancer of the posterior-nasal space include;
(A) A. Proptosis, hearing loss.
(B) B. Hearing loss, double vision.
(C) P
​ urulent nasal discharge, aphonia​.
(D) D. Anosmia, proptosis
Rationale
❏ Aphonia​- inability to speak through disease of or damage to the larynx or mouth.
❏ Proptosis​ can describe any organ that is displaced forward, while exophthalmos
refers to only the eyes. Proptosis can include any directional forward
displacement. Henderson reserves the use of the word exophthalmos for those
cases of proptosis secondary to endocrinological dysfunction
❏ Anosmia​, also known as smell blindness, is the loss of the ability to detect one or
more smells. A ​ nosmia​ may be temporary or permanent. It differs from hyposmia,
which is a decreased sensitivity to some or all smells.
❏ Double vision​ occurs when a person sees a d ​ ouble image​ where there should
only be one. The two images can be side by side, on top of one another, or both.
The condition can affect balance, movement, and reading ability. If d ​ ouble vision
affects just one eye, it is monocular. If it affects both eyes, it is binocular.

(99) during the early postoperative period after a subtotal thyroidectomy, the concern
that has the priority is
(A) hemorrhage
(B) thyrotoxic crisis
(C) airway obstruction
(D) hypocalcemic tetany

(100) immediately after esophageal surgery the priority nursing assessment concerns
the client's
(A) incision
(B) respiration
(C) level of pain
(D) nasogastric tube
Type 5 : Paper 1 trial exam

1. The client presents to the clinic with a serum cholesterol of 275 mg/dL and is placed
on rosuvastatin (Crestor). Which instruction should be given to the client?

A. Report muscle weakness to the physician.


B. Allow six months for the drug to take effect.
C. Take the medication with fruit juice.
D. Ask the doctor to perform a complete blood count before starting the medication.

Rationale
The client taking antilipidemics should be encouraged to report muscle weakness
because this is a sign of rhabdomyolysis. The medication takes effect within 1 month of
beginning therapy, so answer B is incorrect. The medication should be taken with water.
Fruit juice, particularly grapefruit juice, can decrease the drug's effectiveness, so answer
C is incorrect. Liver function studies, not a CBC, should be checked prior to beginning
the medication, so answer D is incorrect

2. The client is admitted to the hospital with hypertensive crises. Diazoxide (Hyperstat)
is ordered. During administration, the nurse should:

A. Utilize an infusion pump


B. Check the blood glucose level
C. Place the client in Trendelenburg position
D. Cover the solution with foil
Rationale
Hyperstat is given IV push for hypertensive crisis. It often causes hyperglycemia. The
glucose level will drop rapidly after the medication is administered. Answer A is
incorrect because this medication is given IV push. The client should be placed in dorsal
recumbent position, not Trendelenburg, so answer C is incorrect. Answer D is incorrect
because the medication is ordered IV push.

3. The 6-month-old client with a ventral septal defect is receiving Digitalis for regulation
of his heart rate. Which finding should be reported to the doctor?

A. Blood pressure of 126/80


B. Blood glucose of 110 mg/dL
C. Heart rate of 60 bpm
D. Respiratory rate of 30 per minute
Rationale
A heart rate of 60 in the 6-month-old receiving Lanoxin elixir (digoxin) should be
reported immediately because bradycardia is associated with digoxin toxicity. The
blood glucose, blood pressure, and respirations are not associated with administration
of Lanoxin, so answers A, B, and D are incorrect.

4. The client admitted with angina is given a prescription for nitroglycerin. The client
should be instructed to:

A. Replenish his supply every 3 months


B. Take one every 15 minutes if pain occurs
C. Leave the medication in the brown bottle
D. Crush the medication and take with water
Rationale
The client should leave the medication in the brown bottle because light deteriorates the
medication. The supply should be replenished every 6 months, so answer A is incorrect.
One tablet should be taken every 5 minutes times three, so answer B is incorrect. If the
pain does not subside, the client should report to the emergency room. The medication
should be taken sublingually and should not be crushed, so answer D is incorrect.

5. The client is instructed regarding foods that are low in fat and cholesterol. Which diet
selection is lowest in saturated fats?

A. Macaroni and cheese


B. Shrimp with rice
C. Turkey breast
D. Spaghetti
Rationale
Turkey contains the least amount of fat and cholesterol. Cheese, shrimp, and beef
should be avoided by the client on a low cholesterol, low fat diet; therefore, answers A,
B, and D are incorrect.

6. The client is admitted with left-sided congestive heart failure. In assessing the client
for edema, the nurse should check the:

A. Feet
B. Neck
C. Hands
D. Sacrum
Rationale
The neck veins should be assessed for distension in the client with congestive heart
failure. Edema of the feet and hands do not indicate central circulatory overload, so
answers A and C are incorrect. Edema of the sacrum is an indication of right-sided
congestive heart failure, so answer D is incorrect.

7. The nurse is checking the client’s central venous pressure. The nurse should place
the zero of the manometer at the:

A. Phlebostatic axis
B. PMI
C. Erb’s point
D. Tail of Spence
Rationale
The nurse should place the zero of the manometer at the phlebostatic axis (located at
the fifth intercostal space midaxillary line) when checking the central venous pressure.
Answers B, C, and D are incorrect methods for determining the central venous pressure.

8. The physician orders lisinopril (Zestril) and furosemide (Lasix) to be administered


concomitantly to the client with hypertension. The nurse should:

A. Question the order


B. Administer the medications
C. Administer separately
D. Contact the pharmacy
Rationale
Zestril is an ACE inhibitor and is frequently given with a diuretic such as Lasix. There is
no need to question the order, give the drugs separately, or contact the pharmacy, so
answers A, C, and D are incorrect.

9. The best method of evaluating the amount of peripheral edema is:

A. Weighing the client daily


B. Measuring the extremity
C. Measuring the intake and output
D. Checking for pitting
Rationale
The best method for evaluating the amount of peripheral edema is measuring the
extremity. A paper tape measure should be used rather than plastic or cloth, and the
area should be marked with a pen. This provides the most objective assessment.
Answers A, C, and D are not the best methods for evaluating the amount of peripheral
edema, therefore they are incorrect.

10. A client with vaginal cancer is being treated with a radioactive vaginal implant. The
client’s husband asks the nurse if he can spend the night with his wife. The nurse
should explain that:

A. Overnight stays by family members is against hospital policy.


B. There is no need for him to stay because staffing is adequate.
C. His wife will rest much better knowing that he is at home.
D. Visitation is limited to 30 minutes when the implant is in place.
Rationale
Clients with radium implants should have close contact limited to 30 minutes per visit.
The general rule is limiting time spent exposed to radium, putting distance between
people and the radium source, and using lead to shield against the radium. Teaching
the family member these principles is extremely important. Answers A, B, and C are not
empathetic and do not address the question; therefore, they are incorrect.

11. The nurse is caring for a client hospitalized with a facial stroke. Which diet selection
would be suited to the client?

A. Roast beef sandwich, potato chips, pickle spear, iced tea


B. Split pea soup, mashed potatoes, pudding, milk
C. Tomato soup, cheese toast, Jello, coffee
D. Hamburger, baked beans, fruit cup, iced tea
Rationale
The client with a facial stroke will have difficulty swallowing and chewing, and the foods
in answer B provide the least amount of chewing. The foods in answers A, C, and D
would require more chewing and, thus, are incorrect

12. The physician has prescribed Novolog insulin for a client with diabetes mellitus.
Which statement indicates that the client knows when the peak action of the insulin
occurs?
A. “I will make sure I eat breakfast within 10 minutes of taking my insulin.”
B. “I will need to carry candy or some form of sugar with me all the time.”
C. “I will eat a snack around three o’clock each afternoon.”
D. “I can save my dessert from supper for a bedtime snack.”
Rationale
Novalog insulin onsets very quickly, so food should be available within 10–15 minutes
of taking the insulin. Answer B does not address a particular type of insulin, so it is
incorrect. NPH insulin peaks in 8–12 hours, so a snack should be eaten at the expected
peak time. It may not be 3 p.m. as stated in answer C. Answer D is incorrect because
there is no need to save the dessert until bedtime.

13. The nurse is teaching basic infant care to a group of first-time parents. The nurse
should explain that a sponge bath is recommended for the first 2 weeks of life because:

A. New parents need time to learn how to hold the baby.


B. The umbilical cord needs time to separate.
C. Newborn skin is easily traumatized by washing.
D. The chance of chilling the baby outweighs the benefits of bathing.
Rationale
The umbilical cord needs time to dry and fall off before putting the infant in the tub.
Although answers A, C, and D might be important, they are not the primary answer to the
question.

14. A client with leukemia is receiving Trimetrexate. After reviewing the client’s chart,
the physician orders Wellcovorin (leucovorin calcium). The rationale for administering
leucovorin calcium to a client receiving Trimetrexate is to:
A. Treat iron-deficiency anemia caused by chemotherapeutic agents
B. Create a synergistic effect that shortens treatment time
C. Increase the number of circulating neutrophils
D. Reverse drug toxicity and prevent tissue damag​e
Rationale
Leucovorin is the antidote for Methotrexate and Trimetrexate which are folic acid
antagonists. Leucovorin is a folic acid derivative. Answers A, B, and C are incorrect
because Leucovorin does not treat iron deficiency, increase neutrophils, or have a
synergistic effect.

15. A 4-month-old is brought to the well-baby clinic for immunization. In addition to the
DPT and polio vaccines, the baby should receive:
A. HibTITER
B. Mumps vaccine
C. Hepatitis B vaccine
D. MMR
Rationale
The Hemophilus influenza vaccine is given at four months with the polio vaccine.
Answers B, C, and D are incorrect because these vaccines are given later in life.

16. The physician has prescribed Nexium (esomeprazole) for a client with erosive
gastritis. The nurse should administer the medication:
A. 30 minutes before meals
B. With each meal
C. In a single dose at bedtime
D. 30 minutes after meals
Rationale
Proton pump inhibitors should be taken prior to the meal. Answers B, C, and D are
incorrect times for giving proton pump inhibitors like Nexium.

17. A client on the psychiatric unit is in an uncontrolled rage and is threatening other
clients and staff. What is the most appropriate action for the nurse to take?

A. Call security for assistance and prepare to sedate the client.


B. Tell the client to calm down and ask him if he would like to play cards.
C. Tell the client that if he continues his behavior he will be punished.
D. Leave the client alone until he calms down.
If the client is a threat to the staff and to other clients the nurse should call for help and
Rationale
Prepare to administer a medication such as Haldol to sedate him. Answer B is incorrect
because simply telling the client to calm down will not work. Answer C is incorrect
because telling the client that if he continues he will be punished is a threat and may
further anger him. Answer D is incorrect because if the client is left alone, he might
harm himself.

18. When the nurse checks the fundus of a client on the first postpartum day, she notes
that the fundus is firm, is at the level of the umbilicus, and is displaced to the right. The
next action the nurse should take is to:
A. Check the client for bladder distention
B. Assess the blood pressure for hypotension
C. Determine whether an oxytocic drug was given
D. Check for the expulsion of small clots
Rationale
If the fundus of the client is displaced to the side, this might indicate a full bladder. The
next action by the nurse should be to check for bladder distention and catheterize, if
necessary. The answers in B, C, and D are actions that relate to postpartum
hemorrhage.

19. A client is admitted to the hospital with a temperature of 99.8°F, complaints of


blood-tinged hemoptysis, fatigue, and night sweats. The client’s symptoms are
consistent with a diagnosis of:

A. Pneumonia
B. Reaction to antiviral medication
C. Tuberculosis
D. Superinfection due to low CD4 count
Rationale
A low-grade temperature, blood-tinged sputum, fatigue, and night sweats are symptoms
consistent with tuberculosis. If the answer in A had said pneumocystis pneumonia,
answer A would have been consistent with the symptoms given in the stem, but just
saying pneumonia isn’t specific enough to diagnose the problem. Answers B and D are
not directly related to the stem.

20. The client is seen in the clinic for treatment of migraine headaches. The drug Imitrex
(sumatriptan succinate) is prescribed for the client. Which of the following in the client’s
history should be reported to the doctor?
A. Diabetes
B. Prinzmetal’s angina
C. Cancer
D. Cluster headaches
Rationale
If the client has a history of Prinzmetal’s angina, he should not be prescribed triptan
preparations because they cause vasoconstriction and coronary spasms. There is no
contraindication for taking triptan drugs in clients with diabetes, cancer, or cluster
headaches, making answers A, C, and D incorrect.

21. The client with suspected meningitis is admitted to the unit. The doctor is
performing an assessment to determine meningeal irritation and spinal nerve root
inflammation. A positive Kernig’s sign is charted if the nurse notes:
A. Pain on flexion of the hip and knee
B. Nuchal rigidity on flexion of the neck
C. Pain when the head is turned to the left side
D. Dizziness when changing positions
Rationale
Kernig’s sign is positive if pain occurs on flexion of the hip and knee. The Brudzinski
reflex is positive if pain occurs on flexion of the head and neck onto the chest so
answer B is incorrect. Answers C and D might be present but are not related to Kernig’s
sign.

22. The client with Alzheimer’s disease is being assisted with activities of daily living
when the nurse notes that the client uses her toothbrush to brush her hair. The nurse is
aware that the client is exhibiting:
A. Agnosia
B. Apraxia
C. Anomia
D. Aphasia
Rationale
Apraxia is the inability to use objects appropriately. Agnosia is loss of sensory
comprehension, anomia is the inability to find words, and aphasia is the inability to
speak or understand, so answers A, C, and D are incorrect.

23. The client with dementia is experiencing confusion late in the afternoon and before
bedtime. The nurse is aware that the client is experiencing what is known as:
A. Chronic fatigue syndrome
B. Normal aging
C. Sundowning
D. Delusions
Rationale
Increased confusion at night is known as “sundowning” syndrome. This increased
confusion occurs when the sun begins to set and continues during the night. Answer A
is incorrect because fatigue is not necessarily present. Increased confusion at night is
not part of normal aging; therefore, answer B is incorrect. A delusion is a firm, fixed
belief; therefore, answer D is incorrect​.

24. The client with confusion says to the nurse, “I haven’t had anything to eat all day
long. When are they going to bring breakfast?” The nurse saw the client in the day room
eating breakfast with other clients 30 minutes before this conversation. Which response
would be best for the nurse to make?
A. “You know you had breakfast 30 minutes ago.”
B. “I am so sorry that they didn’t get you breakfast. I’ll report it to the charge nurse.”
C. “I’ll get you some juice and toast. Would you like something else?”
D. “You will have to wait a while; lunch will be here in a little while.”
Rationale
The client who is confused might forget that he ate earlier. Don’t argue with the client.
Simply get him something to eat that will satisfy him until lunch. Answers A and D are
incorrect because the nurse is dismissing the client. Answer B is validating the delusion.

25. The doctor has prescribed Exelon (rivastigmine) for the client with Alzheimer’s
disease. Which side effect is most often associated with this drug?
A. Urinary incontinence
B. Headaches
C. Confusion
D. Nausea
Rationale
Nausea and gastrointestinal upset are very common in clients taking
acetylcholinesterase inhibitors such as Exelon. Other side effects include l​ iver toxicity,
dizziness, unsteadiness, and clumsiness​. The client might already be experiencing
urinary incontinence or headaches, but they are not necessarily associated; and the
client with Alzheimer’s disease is already confused. Therefore, answers A, B, and C are
incorrect.

26. A client is admitted to the labor and delivery unit in active labor. During examination,
the nurse notes a papular lesion on the perineum. Which initial action is most
appropriate?
A. Document the finding
B. Report the finding to the doctor
C. Prepare the client for a C-section
D. Continue primary care as prescribed
Rationale
Any lesion should be reported to the doctor. This can indicate a herpes lesion. Clients
with open lesions related to herpes are delivered by Cesarean section because there is a
possibility of transmission of the infection to the fetus with direct contact to lesions. It
is not enough to document the finding, so answer A is incorrect. The physician must
make the decision to perform a C-section, making answer C incorrect. It is not enough
to continue primary care, so answer D is incorrect​.

27. A client with a diagnosis of HPV is at risk for which of the following?
A. Hodgkin’s lymphoma
B. Cervical cancer
C. Multiple myeloma
D. Ovarian cancer
Rationale
The client with HPV is at higher risk for cervical and vaginal cancer related to this STI.
She is not at higher risk for the other cancers mentioned in answers A, C, and D, so
those are incorrect.

28. During the initial interview, the client reports that she has a lesion on the perineum.
Further investigation reveals a small blister on the vulva that is painful to touch. The
nurse is aware that the most likely source of the lesion is:
A. Syphilis
B. Herpes
C. Gonorrhea
D. Condylomata
Rationale
A lesion that is painful is most likely a herpetic lesion. A chancre lesion associated with
syphilis is not painful, so answer A is incorrect. Condylomata lesions are painless warts,
so answer D is incorrect. In answer C, gonorrhea does not present as a lesion, but is
exhibited by a yellow discharge.

29. A client visiting a family planning clinic is suspected of having an STI. The best
diagnostic test for treponema pallidum is:
A. Venereal Disease Research Lab (VDRL)
B. Rapid plasma reagin (RPR)
C. Fluorescent treponemal antibody (FTA)
D. Thayer-Martin culture (TMC)
Rationale
Fluorescent treponemal antibody (FTA) is the test for treponema pallidum. VDRL and
RPR are screening tests done for syphilis, so answers A and B are incorrect. The
Thayer-Martin culture is done for gonorrhea, so answer D is incorrect.

30. A 15-year-old primigravida is admitted with a tentative diagnosis of HELLP


syndrome. Which laboratory finding is associated with HELLP syndrome?
A. Elevated blood glucose
B. Elevated platelet count
C. Elevated creatinine clearance
D. Elevated hepatic enzymes
Rationale
The criteria for HELLP is hemolysis, elevated liver enzymes, and low platelet count. In
answer A, an elevated blood glucose level is not associated with HELLP. Platelets are
decreased, not elevated, in HELLP syndrome, as stated in answer B. The creatinine
levels are elevated in renal disease and are not associated with HELLP syndrome, so
answer C is incorrect.

31. The nurse is assessing the deep tendon reflexes of a client with preeclampsia.
Which method is used to elicit the biceps reflex?
A. The nurse places her thumb on the muscle inset in the antecubital space and taps
the thumb briskly with the reflex hammer.
B.​ The nurse loosely suspends the client’s arm in an open hand while tapping the back
of the client’s elbow.
C. The nurse instructs the client to dangle her legs as the nurse strikes the area below
the patella with the blunt side of the reflex hammer.
D. The nurse instructs the client to place her arms loosely at her side as the nurse
strikes the muscle insert just above the wrist.
Rationale
Answer B elicits the triceps reflex, so it is incorrect. Answer C elicits the patellar reflex,
making it incorrect. Answer D elicits the radial nerve, so it is incorrect.

32. A primigravida with diabetes is admitted to the labor and delivery unit at 34 weeks
gestation. Which doctor’s order should the nurse question?
A. Magnesium sulfate 4gm (25%) IV
B. Brethine 10 mcg IV
C. Stadol 1 mg IV push every 4 hours as needed prn for pain
D. Ancef 2gm IVPB every 6 hours
Rationale
Brethine is used cautiously because it raises the blood glucose levels. Answers A, C,
and D are all medications that are commonly used in the diabetic client, so they are
incorrect​.

33. A diabetic multigravida is scheduled for an amniocentesis at 32 weeks gestation to


determine the L/S ratio and phosphatidylglycerol level. The L/S ratio is 1:1 and the
presence of phosphatidylglycerol is noted. The nurse’s assessment of this data is:

A. The infant is at low risk for congenital anomalies.


B. The infant is at high risk for intrauterine growth retardation.
C. The infant is at high risk for respiratory distress syndrome.
D. The infant is at high risk for birth trauma.
Rationale
When the L/S ratio reaches 2:1, the lungs are considered to be mature. The infant will
most likely be small for gestational age and will not be at risk for birth trauma, so
answer D is incorrect. The L/S ratio does not indicate congenital anomalies, as stated in
answer A, and the infant is not at risk for intrauterine growth retardation, making answer
B incorrect.

34. Which observation in the newborn of a diabetic mother would require immediate
nursing intervention?
A. Crying
B. Wakefulness
C. Jitteriness
D. Yawning
Rationale
Jitteriness is a sign of seizure in the neonate. Crying, wakefulness, and yawning are
expected in the newborn, so answers A, B, and D are incorrect.

35. The nurse caring for a client receiving intravenous magnesium sulfate must closely
observe for side effects associated with drug therapy. An expected side effect of
magnesium sulfate is:
A. Decreased urinary output
B. Hypersomnolence
C. Absence of knee jerk reflex
D. Decreased respiratory rate
Rationale
The client is expected to become sleepy, have hot flashes, and be lethargic. A
decreasing urinary output, absence of the knee-jerk reflex, and decreased respirations
indicate toxicity​, so answers A, C, and D are incorrect.

36. The client has elected to have epidural anesthesia to relieve labor pain. If the client
experiences hypotension, the nurse would:
A. Place her in Trendelenburg position
B. Decrease the rate of IV infusion
C. Administer oxygen per nasal cannula
D. Increase the rate of the IV infusion
Rationale
If the client experiences hypotension after an injection of epidural anesthetic, the nurse
should turn her to the left side, apply oxygen by mask, and speed the IV infusion. If the
blood pressure does not return to normal, the physician should be contacted.
Epinephrine should be kept for emergency administration. Answer A is incorrect
because placing the client in Trendelenburg position (head down) will allow the
anesthesia to move up above the respiratory center, thereby decreasing the
diaphragm’s ability to move up and down and ventilate the client. In answer B, the IV
rate should be increased, not decreased. In answer C, the oxygen should be applied by
mask, not cannula.

37. A client has cancer of the pancreas. The nurse should be most concerned about
which nursing diagnosis?
A. Alteration in nutrition
B. Alteration in bowel elimination
C. Alteration in skin integrity
D. Ineffective individual coping
Rationale
Cancer of the pancreas frequently leads to severe nausea and vomiting and altered
glucose levels. The other problems are of lesser concern; thus, answers B, C, and D are
incorrect. Also, according to Maslow’s hierarchy of needs, nutrition is at the most need
as illustrated below
38. The nurse is caring for a client with ascites. Which is the best method to use for
determining early ascites?
A. Inspection of the abdomen for enlargement
B. Bimanual palpation for hepatomegaly
C. Daily measurement of abdominal girth
D. Assessment for a fluid wave

39. The client arrives in the emergency department after a motor vehicle accident.
Nursing assessment findings include BP 80/34, pulse rate 120, and respirations 20.
Which is the client’s most appropriate priority nursing diagnosis?

A. Alteration in cerebral tissue perfusion


B. Fluid volume deficit
C. Ineffective airway clearance
D. Alteration in sensory perception
Rationale
The vital signs indicate hypovolemic shock. They do not indicate cerebral tissue
perfusion, airway clearance, or sensory perception alterations, so answers A, C, and D
are incorrect.

40. The home health nurse is visiting an 18-year-old with osteogenesis imperfecta.
Which information obtained on the visit would cause the most concern? The client:
A. Likes to play football
B. Drinks several carbonated drinks per day
C. Has two sisters with sickle cell tract
D. Is taking acetaminophen to control pain
Rationale
The client with osteogenesis imperfecta is at risk for pathological fractures and is likely
to experience these fractures if he participates in contact sports. Answers B, C, and D
are not factors for concern.

41. The nurse working the organ transplant unit is caring for a client with a white blood
cell count of 450. During evening visitation, a visitor brings a basket of fruit. What action
should the nurse take?
A. Allow the client to keep the fruit
B. Place the fruit next to the bed for easy access by the client
C. Offer to wash the fruit for the client
D. Tell the family members to take the fruit home
Rationale
The client with neutropenia should not have fresh fruit because it should be peeled
and/or cooked before eating. Any source of bacteria should be eliminated, if possible.
Answers A, B, and C will not help prevent bacterial invasions.

42. The nurse is caring for the client following a laryngectomy when suddenly the client
becomes unresponsive and pale, with a BP of 90/40 systolic. The initial nurse’s action
should be to:
A. Place the client in Trendelenburg position
B​. Increase the infusion of Dextrose in normal saline
C. Administer atropine intravenously
D. Move the emergency cart to the bedside
Rationale
The client’s BP is low so increasing the IV is priority. Answers A, C, and D are not the
first priority; therefore, they are incorrect.

43. The client admitted 2 days earlier that a lung resection accidentally pulls out the
chest tube. Which action by the nurse indicates understanding of the management of
chest tubes?
A. Order a chest x-ray
B. Reinsert the tube
C. Cover the insertion site with a Vaseline gauze
D. Call the doctor
Rationale
The nurse’s first action should be to cover the insertion site with an occlusive dressing.
Afterward, the nurse should call the doctor, who will order a chest x-ray and possibly
reinsert the tube. Answers A, B, and D are not the first action to be taken.

44. A client being treated with sodium warfarin has a Protime of 120 seconds. Which
intervention would be most important to include in the nursing care plan?
A. Assess for signs of abnormal bleeding
B. Anticipate an increase in the Coumadin dosage
C. Instruct the client regarding the drug therapy
D. Increase the frequency of neurological assessments
Rationale
The normal Protime is approximately 12–20 seconds. A Protime of 120 seconds
indicates an extremely prolonged Protime and can result in a spontaneous bleeding
episode. Answers B, C, and D may be needed at a later time but are not the most
important actions to take first

45. Which selection would provide the most calcium for the client who is 4 months
pregnant?
A. A granola bar
B. A bran muffin
C. A cup of yogurt
D. A glass of fruit juice
Rationale
The food with the most calcium is the yogurt. Answers A, B, and D are good choices, but
not as good as the yogurt, which has approximately 400mg of calcium.

46. The client with preeclampsia is admitted to the unit with an order for magnesium
sulfate. Which action by the nurse indicates understanding of the possible side effects
of magnesium sulfate?
A. The nurse places a sign over the bed not to check blood pressure in the right arm.
B. The nurse places a padded tongue blade at the bedside.
C. The nurse inserts a Foley catheter.
D. The nurse darkens the room.
Rationale
The client receiving magnesium sulfate should have a Foley catheter in place, and
hourly intake and output should be checked. Answers A, B, and D are incorrect because
they do not indicate understanding of MgSO4 toxicity. Padded tongue blade is no longer
used for seizure precautions
47. A 6-year-old client is admitted to the unit with a hemoglobin of 6g/dL. The physician
has written an order to transfuse 2 units of whole blood. When discussing the
treatment, the child’s mother tells the nurse that she does not believe in having blood
transfusions and that she will not allow her child to have the treatment. What nursing
action is most appropriate?
A. Ask the mother to leave while the blood transfusion is in progress
B. Encourage the mother to reconsider
C. Explain the consequences without treatment
D. N​ otify the physician of the mother’s refusal
Rationale
If the client’s mother refuses the blood transfusion, the doctor should be notified.
Because the client is a minor, the court might order treatment. Answer A is incorrect.
Because it is not the primary responsibility for the nurse to encourage the mother to
consent or explain the consequences, so answers B and C are incorrect.

48. A client is admitted to the unit 2 hours after an explosion causes burns to the face.
The nurse would be most concerned with the client developing which of the following?
A. Hypovolemia
B​. Laryngeal edema
C. Hypernatremia
D. Hyperkalemia
Rationale
The nurse should be most concerned with laryngeal edema because of the area of burn.
The next priority should be answer A, as well as hyponatremia and hypokalemia in C and
D, but these answers are not of primary concern so are incorrect

49. The nurse is evaluating nutritional outcomes for a with anorexia nervosa. Which
data best indicates that the plan of care is effective?
A. The client selects a balanced diet from the menu.
B. The client’s hemoglobin and hematocrit improve.
C. The client’s tissue turgor improves.
D. The client gains weight.
Rationale
The client with anorexia shows the most improvement by weight gain. Selecting a
balanced diet does little good if the client will not eat, so answer A is incorrect. The
hematocrit might improve by several means, such as blood transfusion, but that does
not indicate improvement in the anorexic condition; therefore, answer B is incorrect. The
tissue turgor indicates fluid stasis, not improvement of anorexia, so answer C is
incorrect.

50. The client is admitted following repair of a fractured tibia and cast application.
Which nursing assessment should be reported to the doctor?
A. Pain beneath the cast
B. Warm toes
C. Pedal pulses weak and rapid
D. Paresthesia of the toes
Rationale
At this time, pain beneath the cast is normal. The client’s toes should be warm to the
touch, and pulses should be present. Paresthesia is not normal and might indicate
compartment syndrome. Therefore, Answers A, B, and C are incorrect.

51. A 43-year-old African American male is admitted with sickle cell anemia. The nurse
plans to assess circulation in the lower extremities every 2 hours. Which of the
following outcome criteria would the nurse use?
A. Body temperature of 37.2 celsius or less
B. Toes moved in active range of motion
C. Sensation reported when soles of feet are touched
D. Capillary refill of < 3 seconds
Rationale:
It is important to assess the extremities for blood vessel occlusion in the client with
sickle cell anemia because a change in capillary refill would indicate a change in
circulation. Body temperature, motion, and sensation would not give information
regarding peripheral circulation; therefore, answers A, B, and C are incorrect

52. A 30-year-old male from Haiti is brought to the emergency department in sickle
cell crisis. What is the best position for this client?
A. Side-lying with knees flexed
B. Knee-chest
C. High Fowler's with knees flexed
D. Semi-Fowler's with legs extended on the bed
Rationale
Placing the client in semi-Fowler’s position provides the best oxygenation for this
client. Flexion of the hips and knees, which includes the knee-chest position, impedes
circulation and is not correct positioning for this client. Therefore, answers A, B, and C
are incorrect.
53. A 25-year-old male is admitted in sickle cell crisis. Which of the following
interventions would be of highest priority for this client?
A. Taking hourly blood pressures with mechanical cuff
B. Encouraging​ fluid intake of at least 200mL per hour
C. Position in high Fowler's with knee gatch raised
D. Administering Tylenol as ordered
Rationale
It is important to keep the client in sickle cell crisis hydrated to prevent further sickling
of the blood. Answer A is incorrect because a mechanical cuff places too much
pressure on the arm. Answer C is incorrect because raising the knee gatch impedes
circulation. Answer D is incorrect because Tylenol is too mild an analgesic for the
client in crisis.

54. Which of the following foods would the nurse encourage the client in sickle cell
crisis to eat?
A. Peaches
B. Cottage cheese
C. Popsicle
D. Lima beans
Rationale
Hydration is important in the client with sickle cell disease to prevent thrombus
formation. Popsicles, gelatin, juice, and pudding have high fluid content. The foods in
answers A, B, and D do not aid in hydration and are, therefore, incorrect.

55. A newly admitted client has sickle cell crisis. The nurse is planning care based on
assessment of the client. The client is complaining of severe pain in his feet and
hands. The pulse oximetry is 92. Which of the following interventions would be
implemented first? Assume that there are orders for each intervention.
A. Adjust the room temperature
B. Give a bolus of IV fluids
C. Start O2
D. Administer meperidine (Demerol) 75mg IV push
Rationale :
The pulse oximetry indicates that oxygen levels are low; thus, oxygenation takes
precedence over pain relief. Answer A is incorrect because although a warm
environment reduces pain and minimizes sickling, it would not be a priority. Answer B
is incorrect because although hydration is important, it would not require a bolus.
Answer D is incorrect because Demerol is acidifying to the blood and increases
sickling

56. The nurse is instructing a client with iron-deficiency anemia. Which of the following
meal plans would the nurse expect the client to select?
A. Roast beef, gelatin salad, green beans, and peach pie
B. Chicken salad sandwich, coleslaw, French fries, ice cream
C. Egg salad on wheat bread, carrot sticks, lettuce salad, raisin pie
D. Pork chop, creamed potatoes, corn, and coconut cake
Rationale
Egg yolks, wheat bread, carrots, raisins, and green, leafy vegetables are all high in iron,
which is an important mineral for this client. Roast beef, cabbage, and pork chops are
also high in iron, but the side dishes accompanying these choices are not; therefore,
answers A, B, and D are incorrect.

57. Clients with sickle cell anemia are taught to avoid activities that cause hypoxia and
hypoxemia. Which of the following activities would the nurse recommend?
A. A family vacation in the Rocky Mountains
B. Chaperoning the local boys club on a snow-skiing trip
C. Traveling by airplane for business trips
D. A bus trip to the Museum of Natural History
Rationale
Taking a trip to the museum is the only answer that does not pose a threat. A family
vacation in the Rocky Mountains at high altitudes, cold temperatures, and airplane
travel can cause sickling episodes and should be avoided; therefore, answers A, B,
and C are incorrect.
58. The nurse is conducting an admission assessment of a client with vitamin B12
deficiency. Which of the following would the nurse include in the physical assessment?
A. Palpate the spleen
B. Take the blood pressure
C. Examine the feet for petechiae
D. Examine the tongue
Rationale
The tongue of the client with B12 insufficiency is red and beefy. Answers A, B, and C
incorrect because enlarged spleen, elevated BP, and bradycardia are not associated
with B12 deficiency.

59. An African American female comes to the outpatient clinic. The physician suspects
vitamin B12 deficiency anemia. Because jaundice is often a clinical manifestation of
this type of anemia, what body part would be the best indicator?
A. Conjunctiva of the eye
B. Soles of the feet
C. Roof of the mouth
D. Shins
Rationale
The oral mucosa and hard palate (roof of the mouth) are the best indicators of
jaundice in dark-skinned persons. The conjunctiva can have normal deposits of fat,
which give a yellowish hue; thus, answer A is incorrect. The soles of the feet can be
yellow if they are calloused, making answer B incorrect; the shins would be an area of
darker pigment, so answer D is incorrect.

60. The nurse is conducting a physical assessment on a client with anemia. Which of
the following clinical manifestations would be most indicative of the anemia?
A. BP 146/88
B. Respirations 28 shallow
C. Weight gain of 10 pounds in 6 months
D. Pink complexion
Rationale
When there are fewer red blood cells, there is less hemoglobin and less oxygen.
Therefore, the client is often short of breath, as indicated in answer B. The client with
anemia is often pale in color, has weight loss, and may be hypotensive. Answers A, C,
and D are within normal and, therefore, are incorrect.
61. The nurse is teaching the client with polycythemia vera about prevention of
complications of the disease. Which of the following statements by the client indicates
a need for further teaching?
"I will drink 500mL of fluid or less each day."
"I will wear support hose when I am up."
"I will use an electric razor for shaving."
"I will eat foods low in iron."
Rationale
The client with polycythemia vera is at risk for thrombus formation. Hydrating the
client with at least 3L of fluid per day is important in preventing clot formation, so the
statement to drink less than 500mL is incorrect. Answers B, C, and D are incorrect
because they all contribute to the prevention of complications. Support hose promotes
venous return, the electric razor prevents bleeding due to injury, and a diet low in iron
is essential to preventing further red cell formation.

62. A 33-year-old male is being evaluated for possible acute leukemia. Which of the
following would the nurse inquire about as a part of the assessment?
A. The client collects stamps as a hobby.
B. The client recently lost his job as a postal worker.
C. The client had radiation for treatment of Hodgkin's disease as a teenager.
D. The client's brother had leukemia as a child.
Rationale
Radiation treatment for other types of cancer can contribute to the development of
leukemia. Some hobbies and occupations involving chemicals are linked to leukemia,
but not the ones in these answers; therefore, answers A and B are incorrect. Answer
D is incorrect because the incidence of leukemia is higher in twins, not siblings.

63. An African American client is admitted with acute leukemia. The nurse is
assessing for signs and symptoms of bleeding. Where is the best site for examining
for the presence of petechiae?
A. The abdomen
B. The thorax
C. The earlobes
D. The soles of the feet
Rationale
Petechiae are not usually visualized on dark skin. The soles of the feet and palms of
the hand provide a lighter surface for assessing the client for petechiae. Answers A, B,
and C are incorrect because the skin may be too dark to make an assessment.

64. A client with acute leukemia is admitted to the oncology unit. Which of the
following would be most important for the nurse to inquire?
A. "Have you noticed a change in sleeping habits recently?"
B. "Have you had a respiratory infection in the last 6 months?"
C. "Have you lost weight recently?"
D. "Have you noticed changes in your alertness?"
Rationale
The client with leukemia is at risk for infection and has often had recurrent respiratory
infections during the previous six months. Insomnolence, weight loss, and a decrease in
alertness also occur in leukemia, but bleeding tendencies and infections are the primary
clinical manifestations; therefore, answers A, C, and D are incorrect.

65​. ​Which of the following would be the priority nursing diagnosis for the adult client
with acute leukemia?
A. Oral mucous membrane, altered related to chemotherapy
B. Risk for injury related to thrombocytopenia
C. Fatigue related to the disease process
D. Interrupted family processes related to life-threatening illness of a family
member
Rationale
The client with acute leukemia has bleeding tendencies due to decreased platelet
counts, and any injury would exacerbate the problem. The client would require close
monitoring for hemorrhage, which is of higher priority than the diagnoses in answers
A, C, and D, which are incorrect.

66. A 21-year-old male with Hodgkin's lymphoma is a senior at the local university. He
is engaged to be married and is to begin a new job upon graduation. Which of the
following diagnoses would be a priority for this client?
A. Sexual dysfunction related to radiation therapy
B. Anticipatory grieving related to terminal illness
C. Tissue integrity related to prolonged bed rest
D. Fatigue related to chemotherapy
Rationale
Radiation therapy often causes sterility in male clients and would be of primary
importance to this client. The psychosocial needs of the client are important to address
in light of the age and life choices. Hodgkin’s disease, however, has a good prognosis
when diagnosed early. Answers B, C, and D are incorrect because they are of lesser
priority.

67. A client has autoimmune thrombocytopenic purpura. To determine the client's


response to treatment, the nurse would monitor:
A. Platelet count
B. White blood cell count
C. Potassium levels
D. Partial prothrombin time (PTT)
Rationale
Clients with au​t​oimmune thrombocytopenic purpura (ATP) have low platelet counts,
making answer A the correct answer. White cell counts, potassium levels, and PTT
are not affected in ATP; thus, answers B, C, and D are incorrect.

68.The home health nurse is visiting a client with autoimmune thrombocytopenic


purpura (ATP). The client's platelet count currently is 80, It will be most important to
teach the client and family about:
A. Bleeding precautions
B. Prevention of falls
C. Oxygen therapy
D. Conservation of energy
Rationale
The normal platelet count is 120,000–400,000. Bleeding occurs in clients with low
platelets. The priority is to prevent and minimize bleeding. Oxygenation in answer C is
important, but platelets do not carry oxygen. Answers B and D are of lesser priority
and are incorrect in this instance.

69. A client with a pituitary tumor has had a transsphenoidal hypophysectomy. Which
of the following interventions would be appropriate for this client?
A. Place the client in Trendelenburg position for postural drainage
B. Encourage coughing and deep breathing every 2 hours
C. Elevate the head of the bed 30°
D. Encourage the Valsalva maneuver for bowel movements
Rationale
A prolactinoma is a type of pituitary tumor. Elevating the head of the bed 30° avoids
pressure on the sella turcica and helps to prevent headaches. Answers A, B, and D
are incorrect because Trendelenburg, Valsalva maneuver, and coughing all increase
the intracranial pressure.

70. The client with a history of diabetes insipidus is admitted with polyuria, polydipsia,
and mental confusion. The priority intervention for this client is:
A. Measure the urinary output
B. Check the vital signs
C. Encourage increased fluid intake
D. Weigh the client
Rationale
The large amount of fluid loss can cause fluid and electrolyte imbalance that should be
corrected. The loss of electrolytes would be reflected in the vital signs. Measuring the
urinary output is important, but the stem already says that the client has polyuria, so
answer A is incorrect. Encouraging fluid intake will not correct the problem, making
answer C incorrect. Answer D is incorrect because weighing the client is not
necessary at this time.

71. A client with hemophilia has a nosebleed. Which nursing action is most
appropriate to control the bleeding?
A. Place the client in a sitting position with the head hyperextended
B. Pack the nares tightly with gauze to apply pressure to the source of
bleeding
C. Pinch the soft lower part of the nose for a minimum of 5 minutes
D. Apply ice packs to the forehead and back of the neck
Rationale
C is correct because direct pressure to the nose stops the bleeding. Answers A, B,
and D are incorrect because they do not stop bleeding.

72. A client has had a unilateral adrenalectomy to remove a tumor. To prevent


complications, the most important measurement in the immediate postoperative period
for the nurse to take is:
A. Blood pressure
B. Temperature
C. Output
D. Specific gravity
Rationale
Blood pressure is the best indicator of cardiovascular collapse in the client who has
had an adrenal gland removed. The remaining gland might have been suppressed due
to the tumor activity. Temperature would be an indicator of infection, decreased output
would be a clinical manifestation but would take longer to occur than blood pressure
changes, and specific gravity changes occur with other disorders; therefore, answers
B, C, and D are incorrect.

73. A client with Addison's disease has been admitted with a history of nausea and
vomiting for the past 3 days. The client is receiving IV glucocorticoids (Solu-Medrol).
Which of the following interventions would the nurse implement?
A. Glucometer readings as ordered
B. Intake/output measurements
C. Sodium and potassium levels monitored
D. Daily weights
Rationale
IV glucocorticoids raise the glucose levels and often require coverage with insulin.
Answer B is not necessary at this time, sodium and potassium levels would be
monitored when the client is receiving mineralocorticoids, and daily weights are
unnecessary; therefore, answers B, C, and D are incorrect.

74. A client had a total thyroidectomy yesterday. The client is complaining of tingling
around the mouth and in the fingers and toes. What would the nurses' next action be?
A. Obtain a crash cart
B. Check the calcium level
C. Assess the dressing for drainage
D. Assess the blood pressure for hypertension
Rationale
The parathyroid glands are responsible for calcium production and can be damaged
during a thyroidectomy. The tingling can be due to low calcium levels. The crash cart
would be needed in respiratory distress but would not be the next action to take; thus,
answer A is incorrect. Hypertension occurs in thyroid storms and the drainage would
occur in hemorrhage, so answers C and D are incorrect.

75.A 32-year-old mother of three is brought to the clinic. Her pulse is 52, there is a
weight gain of 30 pounds in 4 months, and the client is wearing two sweaters. The
client is diagnosed with hypothyroidism. Which of the following nursing diagnoses is of
highest priority?
A. Impaired physical mobility related to decreased endurance
B. Hypothermia r/t decreased metabolic rate
C. Disturbed thought processes r/t interstitial edema
D. Decreased cardiac output r/t bradycardia
Rationale
The decrease in pulse can affect the cardiac output and lead to shock, which would
take precedence over the other choices; therefore, answers A, B, and C are incorrect.

76. The nurse is caring for a client hospitalized with a facial stroke. Which diet
selection would be suited to the client?
A. Roast beef sandwich, potato chips, pickle spear, iced tea
B. Split pea soup, mashed potatoes, pudding, milk
C. Tomato soup, cheese toast, Jello, coffee
D. Hamburger, baked beans, fruit cup, iced tea

77. A primigravida, age 42, is 6 weeks pregnant. Based on the client's age, her infant
is at risk for:

A. Down syndrome
B. Respiratory distress syndrome
C. Turner's syndrome
D. Pathological jaundice

78. A client with a missed abortion at 29 weeks gestation is admitted to the hospital.
The client will most likely be treated with:
A. Magnesium sulfate
B. Calcium gluconate
C. Dinoprostone (Prostin E.)
D. Bromocriptine (Pardel)

79. A client with preeclampsia has been receiving an infusion containing magnesium
sulfate for a blood pressure that is 160/80; deep tendon reflexes are 1 plus, and the
urinary output for the past hour is 100mL. The nurse should:

A. Continue the infusion of magnesium sulfate while monitoring the client's blood
pressure
B. Stop the infusion of magnesium sulfate and contact the physician
C. Slow the infusion rate and turn the client on her left side
D. Administer calcium gluconate IV push and continue to monitor the blood
pressure

80. Which statement made by the nurse describes the inheritance pattern of
autosomal recessive disorders?
A. An affected newborn has unaffected parents.
B. An affected newborn has one affected parent.
C. Affected parents have a one in four chance of passing on the defective
gene.
D. Affected parents have unaffected children who are carriers.

81. A pregnant client, age 32, asks the nurse why her doctor has recommended a
serum alpha fetoprotein. The nurse should explain that the doctor has recommended
the test:
A. Because it is a state law
B. To detect cardiovascular defects
C. Because of her age
D. To detect neurological defects
Rationale
Serum alpha-fetoprotein is a blood test to screen for fetal neural tube defects.Alpha
fetoprotein is a screening test done to detect neural tube defects such as spina bifida.
The test is not mandatory, as stated in answer A. It does not indicate cardiovascular
defects, and the mother’s age has no bearing on the need for the test, so answers B
and C are incorrect.

82. A client with hypothyroidism asks the nurse if she will still need to take thyroid
medication during the pregnancy. The nurse's response is based on the knowledge
that:
A. There is no need to take thyroid medication because the fetus's thyroid
produces a thyroid-stimulating hormone.
B. Regulation of thyroid medication is more difficult because the thyroid
gland increases in size during pregnancy.
C. It is more difficult to maintain thyroid regulation during pregnancy due to
a slowing of metabolism.
D. Fetal growth is arrested if thyroid medication is continued during
pregnancy.
Rationale
During pregnancy, the thyroid gland triples in size. This makes it more difficult to
regulate thyroid medication. Answer A is incorrect because there could be a need for
thyroid medication during pregnancy. Answer C is incorrect because the thyroid
function does not slow. Fetal growth is not arrested if thyroid medication is continued,
so answer D is incorrect.

83. The nurse is responsible for performing a neonatal assessment on a full-term


infant. At 1 minute, the nurse would expect to find:
A. An apical pulse of 100
B. An absence of tonus
C. Cyanosis of the feet and hands
D. Jaundice of the skin and sclera
Rationale
Cyanosis of the feet and hands is acrocyanosis. This is a normal finding one minute after birth.
An apical pulse should be 120–160, and the baby should have muscle tone, making answers A
and B incorrect. Jaundice immediately after birth is pathological jaundice and is abnormal, so
answer D is incorrect.

84. A client with sickle cell anemia is admitted to the labor and delivery unit during the
first phase of labor. The nurse should anticipate the client's need for:
A. Supplemental oxygen
B. Fluid restriction
C. Blood transfusion
D. Delivery by Caesarean section
Rationale
Clients with sickle cell crises are treated with heat, hydration, oxygen, and pain relief.
Fluids are increased, not decreased. Blood transfusions are usually not required, and
the client can be delivered vaginally; thus, answers B, C, and D are incorrect.

85. A client with diabetes has an order for ultrasonography. Preparation for an
ultrasound includes:
A. Increasing fluid intake
B. Limiting ambulation
C. Administering an enema
D. Withholding food for 8 hours
Rationale
Before ultrasonography, the client should be taught to drink plenty of fluids and not
void. The client may ambulate, an enema is not needed, and there is no need to
withhold food for eight hours. Therefore, answers B, C, and D are incorrect​.

86. An infant who weighs 8 pounds at birth would be expected to weigh how many
pounds at 1 year?
A. 14 pounds
B. 16 pounds
C. 18 pounds
D. 24 pounds

87. A pregnant client with a history of alcohol addiction is scheduled for a nonstress
test. The nonstress test:
A. Determines the lung maturity of the fetus
B. Measures the activity of the fetus
C. Shows the effect of contractions on the fetal heart rate
D. Measures the neurological well-being of the fetus
Rationale
A nonstress test is done to evaluate periodic movement of the fetus. It is not done to
evaluate lung maturity as in answer A. An oxytocin challenge test shows the effect of
contractions on fetal heart rate and a nonstress test does not measure neurological
well-being of the fetus, so answers C and D are incorrect.

88. A full-term male has hypospadias. Which statement describes hypospadias?


A. The urethral opening is absent.
B. The urethra opens on the dorsal side of the penis.
C. The penis is shorter than usual.
D. The urethra opens on the ventral side of the penis.
Rationale
Hypospadias is a condition in which there is an opening on the underside of the penis.
Answers A, B, and C do not describe hypospadias; therefore, they are incorrect.

89. A gravida III, para II is admitted to the labor unit. Vaginal exam reveals that the
client's cervix is 8cm dilated, with complete effacement. The priority nursing diagnosis
at this time is:
A. Alteration in coping related to pain
B. Potential for injury related to precipitate delivery
C. Alteration in elimination related to anesthesia
D. Potential for fluid volume deficit related to NPO status
Rationale
Transition is the time during labor when the client loses concentration due to intense
contractions. Potential for injury related to precipitate delivery has nothing to do with
the dilation of the cervix, so answer B is incorrect. There is no data to indicate that the
client has had anesthesia or fluid volume deficit, making answers C and D incorrect.

90. The client with varicella will most likely have an order for which category of
medication?
A. Antibiotics
B. Antipyretics
C. Antivirals
D. Anticoagulants
Rationale
Varicella is chicken pox. This herpes virus is treated with antiviral medications. The
client is not treated with antibiotics or anticoagulants, as stated in answers A and D.
The client might have a fever before the rash appears, but when the rash appears, the
temperature is usually gone, so answer B is incorrect.
91. A client is admitted complaining of chest pain. Which of the following drug orders
should the nurse question?
A. Nitroglycerin
B. Ampicillin
C. Propranolol
D. Verapamil
Rationale
Clients with chest pain can be treated with nitroglycerin, a beta blocker such as
propranolol, or Verapamil. There is no indication for an antibiotic such as Ampicillin, so
answers A, C, and D are incorrect.

92. Which of the following instructions should be included in the teaching for the client
with rheumatoid arthritis?
A. Avoid exercise because it fatigues the joints.
B. Take prescribed anti-inflammatory medications with meals.
C. Alternate hot and cold packs to affected joints.
D. Avoid weight-bearing activity.

93. A client with acute pancreatitis is experiencing severe abdominal pain. Which of
the following orders should be questioned by the nurse?
A. Meperidine 100mg IM q 4 hours PRN pain
B. Mylanta 30 ccs q 4 hours via NG
C. Cimetidine 300mg PO q.i.d.
D. Morphine 8mg IM q 4 hours PRN pain

94. The client is admitted to the chemical dependence unit with an order for
continuous observation. The nurse is aware that the doctor has ordered continuous
observation because:
A. Hallucinogenic drugs create both stimulant and depressant effects.
B. Hallucinogenic drugs induce a state of altered perception.
C. Hallucinogenic drugs produce severe respiratory depression.
D. Hallucinogenic drugs induce rapid physical dependence.

95. A client with a history of abusing barbiturates abruptly stops taking the medication.
The nurse should give priority to assessing the client for:
A. Depression and suicidal ideation
B. Tachycardia and diarrhea
C. Muscle cramping and abdominal pain
D. Tachycardia and euphoric mood

96. During the assessment of a laboring client, the nurse notes that the FHT are
loudest in the upper-right quadrant. The infant is most likely in which position?
A. Right breech presentation
B. Right occipital anterior presentation
C. Left sacral anterior presentation
D. Left occipital transverse presentation

97. The primary physiological alteration in the development of asthma is:


A. Bronchiolar inflammation and dyspnea
B. Hypersecretion of abnormally viscous mucus
C. Infectious processes causing mucosal edema
D. Spasm of bronchial smooth muscle

98. A client with mania is unable to finish her dinner. To help her maintain sufficient
nourishment, the nurse should:
A. Serve high-calorie foods she can carry with her
B. Encourage her appetite by sending out for her favorite foods
C. Serve her small, attractively arranged portions
D. Allow her in the unit kitchen for extra food whenever she pleases

99. To maintain Bryant's traction, the nurse must make certain that the child's:
A. Hips are resting on the bed, with the legs suspended at a right angle to
the bed
B. Hips are slightly elevated above the bed and the legs are suspended at a
right angle to the bed
C. Hips are elevated above the level of the body on a pillow and the legs
are suspended parallel to the bed
D. Hips and legs are flat on the bed, with the traction positioned at the foot
of the bed

100. Which action by the nurse indicates understanding of herpes zoster?


A. The nurse covers the lesions with a sterile dressing.
B. The nurse wears gloves when providing care.
C. The nurse administers a prescribed antibiotic.
D. The nurse administers oxygen.

101. The client has an order for a trough to be drawn on the client receiving
Vancomycin. The nurse is aware that the nurse should contact the lab for them to
collect the blood:
A. 15 minutes after the infusion
B. 30 minutes before the infusion
C. 1 hour after the infusion
D. 2 hours after the infusion

102. The client using a diaphragm should be instructed to:


A. Refrain from keeping the diaphragm in longer than 4 hours
B. Keep the diaphragm in a cool location
C. Have the diaphragm resized if she gains 5 pounds
D. Have the diaphragm resized if she has any surgery

103. The nurse is providing postpartum teaching for a mother planning to breastfeed
her infant. Which of the client's statements indicates the need for additional teaching?
A. "I'm wearing a support bra."
B. "I'm expressing milk from my breast."
C. "I'm drinking four glasses of fluid during a 24-hour period."
D. "While I'm in the shower, I'll allow the water to run over my breasts."

104. Damage to the VII cranial nerve results in:


A. Facial pain
B. Absence of ability to smell
C. Absence of eye movement
D. Tinnitus
Rationale
The facial nerve is cranial nerve VII. If damage occurs, the client will experience facial
pain. The auditory nerve is responsible for hearing loss and tinnitus, eye movement is
controlled by the Trochlear or C IV, and the olfactory nerve controls smell; therefore,
answers B, C, and D are incorrect.

105. A client is receiving Pyridium (phenazopyridine hydrochloride) for a urinary tract


infection. The client should be taught that the medication may:
A. Cause diarrhea
B. Change the color of her urine
C. Cause mental confusion
D. Cause changes in taste

106. Which of the following tests should be performed before beginning a prescription
of Accutane?
A. Check the calcium level
B. Perform a pregnancy test
C. Monitor apical pulse
D. Obtain a creatinine level

107. A client with AIDS is taking Zovirax (acyclovir). Which nursing intervention is most
critical during the administration of acyclovir?
A. Limit the client's activity
B. Encourage a high-carbohydrate diet
C. Utilize an incentive spirometer to improve respiratory function
D. Encourage fluids

108. A client is admitted for an MRI. The nurse should question the client regarding:
A. Pregnancy
B. A titanium hip replacement
C. Allergies to antibiotics
D. Inability to move his feet

109. The nurse is caring for the client receiving Amphotericin B. Which of the following
indicates that the client has experienced toxicity to this drug?
A. Changes in vision
B. Nausea
C. Urinary frequency
D. Changes in skin color

110. The nurse should visit which of the following clients first?
A. The client with diabetes with a blood glucose of 95mg/dL
B. The client with hypertension being maintained on Lisinopril
C. The client with chest pain and a history of angina
D. The client with Raynaud's disease

111. A client with cystic fibrosis is taking pancreatic enzymes. The nurse should
administer this medication:
A. Once per day in the morning
B. Three times per day with meals
C. Once per day at bedtime
D. Four times per day

112. Cataracts result in opacity of the crystalline lens. Which of the following best
explains the functions of the lens?
A. The lens controls stimulation of the retina.
B. The lens orchestrates eye movement.
C. The lens focuses light rays on the retina.
D. The lens magnifies small objects.

113. A client who has glaucoma is to have miotic eye drops instilled in both eyes. The
nurse knows that the purpose of the medication is to:
A. Anesthetize the cornea
B. Dilate the pupils
C. Constrict the pupils
D. Paralyze the muscles of accommodation

114. A client with a severe corneal ulcer has an order for Gentamicin gtt. q 4 hours
and Neomycin 1 gtt q 4 hours. Which of the following schedules should be used when
administering the drops?
A. Allow 5 minutes between the two medications.
B. The medications may be used together.
C. The medications should be separated by a cycloplegic drug.
D. The medications should not be used in the same client.

115. The client with color blindness will most likely have problems distinguishing which
of the following colors?
A. Orange
B. Violet
C. Red
D. White

116.The client with a pacemaker should be taught to:


A. Report ankle edema
B. Check his blood pressure daily
C. Refrain from using a microwave oven
D. Monitor his pulse rate
117.The client with enuresis is being taught regarding bladder retraining. The nurse
should advise the client to refrain from drinking after:
A. 1900
B. 1200
C. 1000
D. 0700
118. Which of the following diet instructions should be given to the client with recurring
urinary tract infections?
A. Increase intake of meats.
B. Avoid citrus fruits.
C. Perform pericare with hydrogen peroxide.
D. Drink a glass of cranberry juice every day.
119.The physician has prescribed NPH insulin for a client with diabetes mellitus.
Which statement indicates that the client knows when the peak action of the insulin
occurs?
A. "I will make sure I eat breakfast within 2 hours of taking my insulin."
B. "I will need to carry candy or some form of sugar with me all the time."
C. "I will eat a snack around three o'clock each afternoon."
D. "I can save my dessert from supper for a bedtime snack."
120. A client with pneumocystis carinii pneumonia is receiving trimetrexate. The
rationale for administering leucovorin calcium to a client receiving Methotrexate is to:
A. Treat anemia.
B. Create a synergistic effect.
C. Increase the number of white blood cells.
D. Reverse drug toxicity.
121. A client tells the nurse that she is allergic to eggs, dogs, rabbits, and chicken
feathers. Which order should the nurse question?
A. TB skin test
B. Rubella vaccine
C. ELISA test
D. Chest x-ray
122.The physician has prescribed ranitidine (Zantac) for a client with erosive gastritis.
The nurse should administer the medication:
A. 30 minutes before meals
B. With each meal
C. In a single dose at bedtime
D. 60 minutes after meals
123. A temporary colostomy is performed on the client with colon cancer. The nurse is
aware that the proximal end of a double barrel colostomy:
A. Is the opening on the client's left side
B. Is the opening on the distal end on the client's left side
C. Is the opening on the client's right side
D. Is the opening on the distal right side
124. While assessing the postpartal client, the nurse notes that the fundus is displaced
to the right. Based on this finding, the nurse should:
A. Ask the client to void
B. Assess the blood pressure for hypotension
C. Administer oxytocin
D. Check for vaginal bleeding
125. The physician has ordered an MRI for a client with an orthopedic ailment. An MRI
should not be done if the client has:
A. The need for oxygen therapy
B. A history of claustrophobia
C. A permanent pacemaker
D. Sensory deafness
126. A 6-month-old client is placed on strict bed rest following a hernia repair. Which
toy is best suited to the client?
A. Colorful crib mobile
B. Hand-held electronic games
C. Cars in a plastic container
D. 30-piece jigsaw puzzle
127. The nurse is preparing to discharge a client with a long history of polio. The nurse
should tell the client that:
A. Taking a hot bath will decrease stiffness and spasticity.
B. A schedule of strenuous exercise will improve muscle strength.
C. Rest periods should be scheduled throughout the day.
D. Visual disturbances can be corrected with prescription glasses.
128. A client on the postpartum unit has a proctoepisiotomy. The nurse should
anticipate administering which medication?
A. Dulcolax suppository
B. Docusate sodium (Colace)
C. Methyergonovine maleate (Methergine)
D. Bromocriptine sulfate (Parlodel)

129. A client with pancreatic cancer has an infusion of TPN (Total Parenteral
Nutrition). The doctor has ordered for sliding-scale insulin. The most likely explanation
for this order is:
A. Total Parenteral Nutrition leads to negative nitrogen balance and
elevated glucose levels.
B. Total Parenteral Nutrition cannot be managed with oral hypoglycemics.
C. Total Parenteral Nutrition is a high-glucose solution that often elevates
the blood glucose levels.
D. Total Parenteral Nutrition leads to further pancreatic disease.

130. An adolescent primigravida who is 10 weeks pregnant attends the antepartal


clinic for a first check-up. To develop a teaching plan, the nurse should initially assess:
A. The client's knowledge of the signs of preterm labor
B. The client's feelings about the pregnancy
C. Whether the client was using a method of birth control
D. The client's thought about future children

131.An obstetric client is admitted with dehydration. Which IV fluid would be most
appropriate for the client?
A. 0.45 normal saline
B. Dextrose 1% in water
C. Lactated Ringer's
D. Dextrose 5% in 0.45 normal saline

132. The physician has ordered a thyroid scan to confirm the diagnosis. Before the
procedure, the nurse should:
A. Assess the client for allergies
B. Bolus the client with IV fluid
C. Tell the client he will be asleep
D. Insert a urinary catheter

133. The physician has ordered an injection of RhoGam for a client with blood type A
negative. The nurse understands that RhoGam is given to:
A. Provide immunity against Rh isoenzymes
B. Prevent the formation of Rh antibodies
C. Eliminate circulating Rh antibodies
D. Convert the Rh factor from negative to positive

134. The nurse is caring for a client admitted to the emergency room after a fall.
X-rays reveal that the client has several fractured bones in the foot. Which treatment
should the nurse anticipate for the fractured foot?
A. Application of a short inclusive spica cast
B. Stabilization with a plaster-of-Paris cast
C. Surgery with Kirschner wire implantation
D. A gauze dressing only

135. A client with bladder cancer is being treated with iridium seed implants. The
nurse's discharge teaching should include telling the client to:
A. Strain his urine
B. Increase his fluid intake
C. Report urinary frequency
D. Avoid prolonged sitting

136. Following a heart transplant, a client is started on medication to prevent organ


rejection. Which category of medication prevents the formation of antibodies against
the new organ?
A. Antivirals
B. Antibiotics
C. Immunosuppressants
D. Analgesics

137. The nurse is preparing a client for cataract surgery. The nurse is aware that the
procedure will use:
A. Mydriatics to facilitate removal
B. Miotic medications such as Timoptic
C. A laser to smooth and reshape the lens
D. Silicone oil injections into the eyeball

138. A client with Alzheimer's disease is awaiting placement in a skilled nursing


facility. Which long-term plans would be most therapeutic for the client?
A. Placing mirrors in several locations in the home
B. Placing a picture of herself in her bedroom
C. Placing simple signs to indicate the location of the bedroom, bathroom,
and so on
D. Alternating healthcare workers to prevent boredom

139. A client with an abdominal cholecystectomy returns from surgery with a


Jackson-Pratt drain. The chief purpose of the Jackson-Pratt drain is to:
A. Prevent the need for dressing changes
B. Reduce edema at the incision
C. Provide for wound drainage
D. Keep the common bile duct open
140. The nurse is performing an initial assessment of a newborn Caucasian male
delivered at 32 weeks gestation. The nurse can expect to find the presence of:
A. Mongolian spots
B. Scrotal rugae
C. Head lag
D. Vernix caseosa

141. The nurse is caring for a client admitted with multiple trauma. Fractures include
the pelvis, femur, and ulna. Which finding should be reported to the physician
immediately?
A. Hematuria
B. Muscle spasms
C. Dizziness
D. Nausea

142. A client is brought to the emergency room by the police. He is combative and
yells, "I have to get out of here. They are trying to kill me." Which assessment is most
likely correct in relation to this statement?
A. The client is experiencing an auditory hallucination.
B. The client is having a delusion of grandeur.
C. The client is experiencing paranoid delusions.
D. The client is intoxicated.

143. The nurse is preparing to suction the client with a tracheotomy. The nurse notes
a previously used bottle of normal saline on the client's bedside table. There is no
label to indicate the date or time of initial use. The nurse should:
A. Lip the bottle and use a pack of sterile 4x4 for the dressing
B. Obtain a new bottle and label it with the date and time of first use
C. Ask the ward secretary when the solution was requested
D. Label the existing bottle with the current date and time

144. An infant's Apgar score is 9 at 5 minutes. The nurse is aware that the most likely
cause for the deduction of one point is:
a. The baby is cold.
b. The baby is experiencing bradycardia.
c. The baby's hands and feet are blue.
d. The baby is lethargic.

145. The primary reason for rapid continuous rewarming of the area affected by
frostbite is to:
A. Lessen the amount of cellular damage
B. Prevent the formation of blisters
C. Promote movement
D. Prevent pain and discomfort

146. A client recently started on hemodialysis wants to know how the dialysis will take
the place of his kidneys. The nurse's response is based on the knowledge that
hemodialysis works by:
A. Passing water through a dialyzing membrane
B. Eliminating plasma proteins from the blood
C. Lowering the pH by removing nonvolatile acids
D. Filtering waste through a dialyzing membrane

147. During a home visit, a client with AIDS tells the nurse that he has been exposed
to measles. Which action by the nurse is most appropriate?

a. Administer an antibiotic
b. Contact the physician for an order for immune globulin
c. Administer an antiviral
d. Tell the client that he should remain in isolation for 2 weeks

150. A client hospitalized with MRSA (methicillin-resistant staph aureus) is placed on


contact precautions. Which statement is true regarding precautions for infections
spread by contact?
A. The client should be placed in a room with negative pressure.
B. Infection requires close contact; therefore, the door may remain open.
C. Transmission is highly likely, so the client should wear a mask at all
times.
D. Infection requires skin-to-skin contact and is prevented by hand washing,
gloves, and a gown.

151. A client who is admitted with an above-the-knee amputation tells the nurse that
his foot hurts and itches. Which response by the nurse indicates understanding of
phantom limb pain?
A. "The pain will go away in a few days."
B. "The pain is due to peripheral nervous system interruptions. I will get you
some pain medication."
C. "The pain is psychological because your foot is no longer there."
D. "The pain and itching are due to the infection you had before the
surgery."

152. A client with cancer of the pancreas has undergone a Whipple procedure. The
nurse is aware that during the Whipple procedure, the doctor will remove the:
A. Head of the pancreas
B. Proximal third section of the small intestines
C. Stomach and duodenum
D. Esophagus and jejunum
Rationale
During a Whipple procedure the head of the pancreas, which is a part of the stomach,
the jejunum, and a portion of the stomach are removed and anastomosed. Answer B
is incorrect because the proximal third of the small intestine is not removed. The entire
stomach is not removed, as in answer C, and in answer D, the esophagus is not
removed.

153. The physician has ordered a minimal-bacteria diet for a client with neutropenia.
The client should be taught to avoid eating:
A. Fruits
B. Salt
C. Pepper
D. Ketchup
Rationale
Pepper is not processed and contains bacteria. Answers A, B, and D are incorrect
because fruits should be cooked or washed and peeled, and salt and ketchup are
allowed.

154. A client is discharged home with a prescription for Coumadin (sodium warfarin).
The client should be instructed to:
A. Have a Protime done monthly
B. Eat more fruits and vegetables
C. Drink more liquids
D. Avoid crowds
Rationale
Coumadin is an anticoagulant. One of the tests for bleeding time is a Protime. This
test should be done monthly. Eating more fruits and vegetables is not necessary, and
dark-green vegetables contain vitamin K, which increases clotting, so answer B is
incorrect. Drinking more liquids and avoiding crowds is not necessary, so answers C
and D are incorrect.

155. The nurse is assisting the physician with removal of a central venous catheter. To
facilitate removal, the nurse should instruct the client to:
A. Perform the Valsalva maneuver as the catheter is advanced
B. Turn his head to the left side and hyperextend the neck
C. Take slow, deep breaths as the catheter is removed
D. Turn his head to the right while maintaining a sniffing position
Rationale
The client who is having a central venous catheter removed should be told to hold his
breath and bear down. This prevents air from entering the line. Answers B, C, and D
will not facilitate removal.

156. A client has an order for streptokinase. Before administering the medication, the
nurse should assess the client for:
A. Allergies to pineapples and bananas
B. A history of streptococcal infections
C. Prior therapy with phenytoin
D. A history of alcohol abuse
Rationale
Clients with a history of streptococcal infections could have antibodies that render the
streptokinase ineffective. There is no reason to assess the client for allergies to
pineapples or bananas, there is no correlation to the use of phenytoin and
streptokinase, and a history of alcohol abuse is also not a factor in the order for
streptokinase; therefore, answers A, C, and D are incorrec​t.

157. The nurse is providing discharge teaching for the client with leukemia. The client
should be told to avoid:
A. Using oil- or cream-based soaps
B. Flossing between the teeth
C. The intake of salt
D. Using an electric razor
Rationale
The client who is immune-suppressed and has bone marrow suppression should be
taught not to floss his teeth because platelets are decreased. Using oils and
cream-based soaps is allowed, as is eating salt and using an electric razor; therefore,
answers A, C, and D are incorrect.

158. The nurse is changing the ties of the client with a tracheotomy. The safest
method of changing the tracheostomy ties is to:
A. Apply the new tie before removing the old one.
B. Have a helper present.
C. Hold the tracheotomy with the nondominant hand while removing the old
tie.
D. Ask the doctor to suture the tracheostomy in place.
Rationale
The best method and safest way to change the ties of a tracheotomy is to apply the
new ones before removing the old ones. Having a helper is good, but the helper might
not prevent the client from coughing out the tracheotomy. Answer C is not the best
way to prevent the client from coughing out the tracheotomy. Asking the doctor to
suture the tracheostomy in place is not appropriate.

159. The nurse is monitoring a client following a lung resection. The hourly output from
the chest tube was 300mL. The nurse should give priority to:
A. Turning the client to the left side
B. Milking the tube to ensure patency
C. Slowing the intravenous infusion
D. Notifying the physician
Rationale
The output of 300mL is indicative of hemorrhage and should be reported immediately.
Answer A does nothing to help the client. Milking the tube is done only with an order
and will not help in this situation, and slowing the intravenous infusion is not correct;
thus, answers B and C are incorrect.

160. The infant is admitted to the unit with tetralogy of fallot. The nurse would
anticipate an order for which medication?
a. Digoxin
b. Epinephrine
c. Aminophylline
d. Atropine
Rationale
The infant with tetralogy of Fallot has four heart defects. He will be treated with digoxin
to slow and strengthen the heart. Epinephrine, aminophylline, and atropine will speed
the heart rate and are not used in this client; therefore, answers B, C, and D are
incorrect.
161. The toddler is admitted with a cardiac anomaly. The nurse is aware that the infant
with a ventricular septal defect will:
A. Tire easily
B. Grow normally
C. Need more calories
D. Be more susceptible to viral infections
Rationale
The toddler with a ventricular septal defect will tire easily. He will not grow normally but
will not need more calories. He will be susceptible to bacterial infection, but he will be
no more susceptible to viral infections than other children. Therefore, answers B, C,
and D are incorrect.
162. The nurse is monitoring a client with a history of stillborn infants. The nurse is
aware that a nonstress test can be ordered for this client to:
A. Determine lung maturity
B. Measure the fetal activity
C. Show the effect of contractions on fetal heart rate
D. Measure the well-being of the fetus
Rationale
A nonstress test determines periodic movement of the fetus. It does not determine
lung maturity, show contractions, or measure neurological well-being, making answers
A, C, and D incorrect.
163. A client with clotting disorder has an order to continue Lovenox (enoxaparin)
injections after discharge. The nurse should teach the client that Lovenox injections
should:
A. Be injected into the deltoid muscle
B. Be injected into the abdomen
C. Aspirate after the injection
D. Clear the air from the syringe before injections
Rationale
Lovenox injections should be given in the abdomen, not in the deltoid muscle. The
client should not aspirate after the injection or clear the air from the syringe before
injection. Therefore, answers A, C, and D are incorrect.
164. The nurse has a preop order to administer Valium (diazepam) 10mg and
Phenergan (promethazine) 25mg. The correct method of administering these
medications is to:
A. Administer the medications together in one syringe
B. Administer the medication separately
C. Administer the Valium, wait 5 minutes, and then inject the Phenergan
D. Question the order because they cannot be given at the same time
Rationale
Valium is not given in the same syringe with other medications, so answer A is
incorrect. These medications can be given to the same client, so answer D is
incorrect. In answer C, it is not necessary to wait to inject the second medication.
Valium is an antianxiety medication, and Phenergan is used as an antiemetic.
165. The client has recently returned from having a thyroidectomy. The nurse should
keep which of the following at the bedside?
A. A​ ​tracheotomy set
B. A padded tongue blade
C. An endotracheal tube
D. An airway
Rationale
The client who has recently had a thyroidectomy is at risk for tracheal edema. A
padded tongue blade is used for seizures and not for the client with tracheal edema,
so answer B is incorrect. If the client experiences tracheal edema, the endotracheal
tube or airway will not correct the problem, so answers C and D are incorrect.

166. The physician has ordered a histoplasmosis test for the elderly client. The nurse
is aware that histoplasmosis is transmitted to humans by:
A. Cats
B. Dogs
C. Turtles
D. Birds
Rationale
Histoplasmosis is a fungus carried by birds. It is not transmitted to humans by cats,
dogs, or turtles. Therefore, answers A, B, and C are incorrect​.​Histoplasmosis is a type
of lung infection. It is caused by inhaling Histoplasma capsulatum fungal spores.
These spores are found in soil and in the droppings of bats and birds. This fungus
mainly grows in the central, southeastern, and mid-Atlantic states.

https://www.pearsonitcertification.com/articles/article.aspx?p=2101528&seqNum=2
https://www.pearsonitcertification.com/articles/article.aspx?p=402459&seqNum=2

You might also like